Download as docx, pdf, or txt
Download as docx, pdf, or txt
You are on page 1of 51

ASTIGNURSE ONLINE TUTORIAL REVIEW

All rights are reserved. No part of this publication may be reproduced, stored in a
retrieval system or transmitted in any form or by any means, electronic,
mechanical, photocopying, recording or otherwise, without prior permission.

500 MIDWIFERY BOARD EXAM POINTERS

OBSTETRICS

1. The temporary opening between the two atria of the fetal heart that is expected to close after birth
of the baby is ____________.
A. ductus arteroisus C. ductus venosus
B. foramen ovale D. aorta

2. The following are recorded in the partograph, EXCEPT:


A. fetal heart beat C. uterine contractions
B. height of the uterine fundus D. amniotic fluid

3. One of the following conditions predisposes to subinvolution:


A. retained placental tissues C. labor of 8 hours
B. pre-eclampsia D. urinary tract infection

4. When the bag of the water breaks and the umbilical cord prolapses, the proper step to do is
________.
A. Assume Fowler's position C. Push the cord inside the vagina
B. Push fetal head upward to relieve pressure on cord D. Stand up and walk hurry labor

5. During nutrition counseling of a pregnant woman:


A. recommend that she weigh herself twice a week.
B. inquire what she eats in a typical day to determine if her diet is adequate.
C. advice her that only very anemic women need iron/folate supplements.
D. tell her to eat the same amount of food that she ate before her pregnancy.

6. Using the WHO partograph, the internal examination was 4cm at 7AM. If labor will progress and
cervix will dilate at one cm per hour, full cervical dilation is expected to occur at _________.
A. 11:00 AM C. 1:00 PM
B. 2:00 PM D. 12:00 NOON

7. The presence of meconium stained amniotic fluid is common in breech presentation during labor
and is considered ______________.
A. a sign to hasten child delivery C. an indication for cesarean section always
B. an indication for referral to the hospital D. always a sign of fetal distress

8. In an emergency, the appropriate intravenous fluid to use for infusion:


A. Dextrose in 10% water C. D5 normal saline solution
B. Ringer's lactate solution D. Dextrose in 5% water

9. One of the following is considered proper care of the second degree perineal lacerations wound:
A. Keep the perineum clean and dry C. Remove absorbable sutures 5 days postpartum
B. Rest and lie in bed for 2 days D. Put patient on liquid diet for 2 days

10. Labor contractions in G2P1 is regular and now in active phase of labor. Which of the following
findings will prompt you to refer the woman to the hospital?
A. five cm dilation for four hours C. one cm dilation for three hours

1
B. two cm dilation for two hours D. four cm dilation in three hours

11. The baby was delivered one hour ago. The placenta has not yet been expelled. The correct next
step that the midwife should do is __________.
A. push the uterus downwards C. manually remove the placenta
B. pull umbilical cord outward D. refer the woman to hospital

12. A 22 year-old primigravida at 37 weeks age of gestation has epigastric pain, blurred vision and
severe headache. These danger signs are most likely due to ____________.
A. Migraine C. hypertension
B. peptic ulcer D. hematoma

13. The fundal height measurement may not reflect the baby's progressive growth if the presentation
of the fetus is ___________.
A. Brow C. frank breech
B. Shoulder D. cephalic

14. A grande multipara is a woman with more than:______


A. three deliveries C. five deliveries
B. two deliveries D. four deliveries

15. If a normal cervix is 2cm long, when it become only 1 cm in length, it is:______
A. 75% effaced C. 100% effaced
B. 25% effaced D. 50% effaced

Situation - Ella, a 26 years old G2P1 on her 12 weeks age of gestation came to the health care center for
antenatal care. Her Body Mass Index (BMI) is 21. Her previous baby was born term with a birth weight of
2450 grams. You will advice her on proper nutrition during pregnancy to have a better newborn outcome.

16. Based on her Body Mass Index, Ella should have a target weight gain of ________kg during the
duration of this pregnancy.
A. 5 kg - 9 kg C. 12.5 kg - 18 kg
B. 11.5 kg - 12.5 kg D. 7 kg - 11.5 kg

17. One of the following statements concerning Vitamin A intake during pregnancy is INCORRECT.
A. a good source of this vitamin is sunlight.
B. alcohol intake can enhance toxicity of Vitamin A.
C. doses of >25,000 IU is teratogenic.
D. this vitamin helps prevent night blindness.

18. The statement regarding nutrition intake of the pregnant women that is ACCURATE is
_________.
A. excessive intake of caffeine has no deleterious effect on the newborn
B. balance energy and protein supplementation benefit undernourished women
C. vitamins E and C are essential in decreasing preterm and post term births
D. routine calcium supplementation is proven to benefit all pregnant women

19. The risk factors for gestational diabetes mellitus during pregnancy that put the well-being of the
mother and baby in danger, are the following, EXCEPT _____________.
A. family history of hypertension C. obese women
B. previous macrosomia D. family history of diabetes

20. The statement regarding anemia during pregnancy that is CORRECT is _____________.
A. The dose of folic acid supplement is 0.4 grams daily
B. Dose for anemic pregnant women is 30 mg daily
C. Third trimester hemoglobin is more than>11 g/L

2
D. Correcting anemia prevents sepsis and Low birth weigh t

Situation - Pelvic infections presenting as vaginal discharge and lesions is among the problems noted by
women. The following are scenarios related to these conditions.

21. Susan has repeated episodes of Pelvic Inflammatory Disease (PID). This can result to ectopic
pregnancy because the pelvic organ most often affected is the ______________.
A. Ovaries C. oviduct
B. Uterus D. vagina

22. This 33 years-old G5P5 came because of reddish vulva, vaginal itchiness and cheesy-white
discharge. The appropriate examination to do to confirm the diagnosis is ___________.
A. KOH mount C. hanging drop
B. acetic acid wash D. Gram stain

23. Which one of the following statements regarding genital Herpes is correct?
A. Effectively treated with antibiotics.
B. It is a short term infection with bacteria.
C. Generally presents as yellowish vaginal discharge.
D. Appears as vesicular or ulcerative lesions in vulva.

24. In Chlamydia trachomatis infection in pregnant women, drugs like Tetracycline, should not be
taken because of harm the fetus resulting to __________.
A. discolored teeth C. atrial septal defect
B. pulmonary hypoplasia D. congenital blindness

25. The sexually transmitted microbe that is responsible for Hutchinson's teeth and chancre is
______________.
A. Condyloma acuminata C. Trichomonas vaginalis
B. Syphilis D. Human Immuno Deficiency Virus

Situation - Fanny 22 years-old, G1P0 wants some information regarding her current pregnancy. Her
concerns are on the body changes and uterine growth that she noted when she became pregnant, and
needs some information on them.

26. On abdominal examination you feel that the uterine fundus is palpable just above the symphysis
pubis. Based on the uterine size, the age of gestation is approximately _______ weeks.
A. 16 C. 12
B. 20 D. 8

27. Quickening is expected to be felt by Fanny on her ___________ weeks of pregnancy.


A. 20 C. 12
B. 24 D. 16

28. Due to the effect of the hormone progesterone, Fanny may feel discomfort during pregnancy like
A. Headache C. constipation
B. Palpitation D. nausea

29. Regular intake of iron tablets is beneficial but may produce effects like ___________.
A. hypogastric pain happens often C. constipation happen less often
B. stool will have dark/black color D. stools becomes bloody red

30. The following changes observed by the pregnant woman are considered normal, EXCEPT ____.
A. Braxton- Hicks contractions C. Hyperemesis gravidarum
B. colostrum production D. striae gravidarum

3
Situation - The registered midwife is assigned to monitor labor at the municipal birthing home. She is
expected to have a basic knowledge of the pelvic anatomy to assess the passage of the baby and allow a
successful vaginal delivery.

31. The normal measurement of the interspinous diameter is __________.


A. 8 cm C. 13.5 cm
B. 10 cm D. 12 cm

32. The anterior border of the brim of the true pelvis is the ____________.
A. promontory of sacrum C. innominate bone
B. superior ramus pubic bone D. symphysis pubis

33. Prolonged and obstructed labor may result to having an abnormal connection between the vagina
and urinary bladder called ___________.
A. vaginal prolapse C. rectal prolapse
B. rectovaginal fistula D. vesicovaginal fistula

34. When the body of the uterus leans backwards to the rectum, this position is called _________.
A. Anteversion C. retroflexion
B. Anteflexion D. retroversion

35. The anatomic fixed reference point to determine the station of the fetal head is/are the _____
A. ischial spines C. sacrum
B. coccyx D. ilium

Situation - The midwife is expected to know the procedures and interventions that are beneficial for the
pregnant woman and to avoid those that are ineffective that may result to harm.

36. The following practices are considered effective/beneficial form of care EXCEPT __________.
A. restrict salt to prevent pre- eclampsia C. taking ferrous sulfate to lessen anemia
B. use of magnesium sulfate for eclampsia D. take antibiotics to treat bacteriuria

37. One beneficial and evidence-based clinical practice for the care of woman during labor is ____
A. intravenous fluids should be routinely given.
B. maintain the woman on Nothing Per Orem
C. start plotting the partograph at 3cm dilation
D. allow the woman to ambulate and drink liquid

38. Routine injection of oxytocin postpartum a beneficial form of care because it __________.
A. decrease postpartum psychosis C. lessen nausea and vomiting
B. lessens postpartum bleeding D. decrease venous thromboembolism

39. One of the following statements regarding viral hepatitis infection is true, EXCEPT __________.
A. common cause of jaundice in pregnancy C. may be transmitted across the placenta
B. positive HBsAg means chronic carrier D. hepatitis B is a food borne infection

40. A primigravida whose LMP was February 8, 2019 delivered her baby on September 9, 2019. The
immediate danger to the newborn with birth at this AOG is __________.
A. tetanus neonatorum C. hyaline membrane disease
B. congenital deformity D. post maturity syndrome
RATIONALE: Based on the situation, the baby was born premature. The baby is born 7 months so the
baby is high risk for hyaline membrane disease

Situation - Mely, 26 years old G2P1 with a 39 weeks gestation was admitted at the birthing center
because of true labor contractions. Her vital signs were all within normal. Fundal height is 32 cm. Fetal

4
heart beat is at 150 bpm. She now has a 6 cm cervical dilation and an engaged fetal head. You will now
begin labor watch and do monitoring using the partograph.

41. Based on the internal examination findings, Mely is now on the _________ phase of labor.
A. pre – latent C. active
B. latent D. protracted

42. Plotting the partograph, the "X" mark should be placed at the intersection between the time of
examination and cervical dilation at the __________ line.
A. Alert C. horizontal
B. Vertical D. action

43. After four hours of good uterine contractions, a repeat vaginal examination was done. With good
progress happening of one centimeter per hour, cervical dilation is now expected to be at __cm.
A. 8 C. 7
B. 9 D. 10

44. The total time of Mely's labor is 5 hours beginning from the time of her first internal examination
(including one hour 2nd stage) . Based on this finding, we can conclude that the length of her
labor is ____________ .
A. Abnormal C. normal
B. Precipitate D. prolonged

45. In assessing progress, when labor goes beyond the right of action line in the partograph without
enough cervical dilation, this means that this woman's labor is _____________
A. normal pattern, just wait C. progressing, do not to worry
B. progressing as expected D. poor progress, for referral

Situation - Rosario, 26 years old G3P2 on her 39 weeks age of pregnancy came to the birthing center
because of labor contractions. She claims to have been in labor for the past 5 hours at home describing
that the pains are now located on her lower abdomen and the lower back too. Her fundal height is 32 cm
and her previous baby's weights are about 2800 - 2900 grams. You will now perform examination on this
pregnant women.

46. One of the following is NOT an indication for doing internal examination:
A. Check the baby's presentation C. Assess her cervical dilation
B. Confirm placenta previa D. Confirm rupture of membranes

47. On examination, you noted that there is Non-Engagement of the fetal head in the maternal pelvis
in this 39 weeks pregnancy. The most likely explanations is __________.
A. big baby for the pelvic diameter C. transverse pelvic diameter of 15 cm
B. well formed lower uterine segment D. well flexed & positioned fetal head

48. When the mento-vertical diameter of the baby presents in the birth canal, the part of fetus that is
delivered first is the _____________.
A. Chin C. face
B. Occiput D. breech

49. On internal examination, the acromion of the baby lies on the birth canal, the presentation of the
baby is _______________.
A. Shoulder C. brow
B. complete breech D. incomplete breech

50. Engagement of the baby in the birth canal has occurred because the ___________.
A. bitemporal diameter has passed pelvic brim in cephalic presentation

5
B. widest presenting diameter has passed through the brim of the pelvis
C. fetal knees has passed through the pelvic brim in breech presentation
D. baby's shoulder blade is now at the brim of the maternal pelvic canal

Situation - At the health center, you are the midwife assigned to do antenatal care. The following are
questions that you need to when the patient asks them.

51. The procedure or examination that is the surest evidence that a woman with missed period is
pregnant is _________.
A. abdominal palpitation C. pregnancy test
B. ultrasound D. uterine enlargement

52. Regarding dental problems and issues during pregnancy, which of the following statements is
TRUE?
A. tooth loss happens more often C. gums easily bleed in pregnancy
B. tooth extraction is not allowed D. dental fillings are not allowed

53. She complains of darkening of the skin around the areola and linea nigra. She wants these
darkened areas to fade away. You will advise her to ____________.
A. scrub the area with scented soap C. inject glutathione whitening soap
B. wait as it will surely vanish after birth D. accept as normal hormonal effect

54. The increase in vaginal secretion during pregnancy bothers her. After ruling out infection, you
concluded that this condition is likely due to ___________.
A. poor or lack personal hygiene C. effect of pregnancy hormones
B. cured by scented feminine wash D. use of synthetic fibers underwear

55. On pelvic ultrasound, in a full-term pregnancy with intact bag of water, when the volume of
amniotic fluid is very low, the condition that is likely present is ____________.
A. shiny abdominal skin C. large fundal height
B. small fundal height D. fetus urinates well

Situation - Cora G1P0, 21 years old came to the health center because of 9 weeks missed period. She is
very sure of her last menstrual period because her menses come regularly. She complains of having
nausea and vomiting and frequently urination.

56. The absence of menstruation is a __________ sign of pregnancy.


A. Plausible C. probable
B. Positive D. presumptive

57. Which of the following examinations is NOT routinely requested during the woman's prenatal
care?
A. Blood platelet count C. Hepatitis B surface antigen
B. Complete blood count D. Routine urinalysis

58. On abdominal examination, based on Cora's age of gestation, the uterine fundus is expected to
be located _________
A. between symphysis and navel C. at the level of the umbilicus
B. just above the umbilicus D. just below the symphysis

RATIONALE: At 9 weeks, the uterus is still in the pelvic cavity, the fundic height is located below
symphysis pubis.

59. In order to protect her and her baby against clostridium-borne infection, especially at the time
of birth, the vaccine that should be injected twice during pregnancy is ___________.
A. Oral Polio C. Tetanus toxoid

6
B. Pneumonia D. MMR vaccine

60. A reactive hepatitis B surface antigen means that the woman has a/an __________.
A. history of hepatitis B infection C. previous hepatitis A infection
B. infectious condition at present D. active hepatitis B infection

Situation - Veronica, 28 years old G4P2 consulted because of vaginal spotting of a few hours duration
with mild hypogastric pains. Her last menstrual period was 12 weeks ago. She had a previous
miscarriage. A pregnancy test done two weeks ago was positive.

61. If her cervix is closed, non-tender without passage of tissues, the likely diagnosis is _________
abortion.
A. Missed C. complete
B. Incomplete D. threatened

62. The back-up doctor requested pelvic ultrasound with the findings of "intrauterine pregnancy
without cardiac activity" . Veronica had moderate and continuous bleeding and this time with an
open cervix. The likely diagnosis now is ___________ abortion.
A. Missed C. threatened
B. Incomplete D. complete

63. The definite management for this type of abortion with open cervix and retained tissue is _____.
A. progesterone tablets C. isoxsuprine tablets
B. observation and rest D. completion curettage

64. On her follow-up visit one week after her hospitalization, she was noted to have a pale palms and
a hemoglobin of 9 grams/dl Which of the following medicines will NOT be beneficial to her
condition?
A. ferrous sulphate C. ferrous gluconate
B. magnesium sulphate D. ferrous fumarate

65. Now, decided on the long acting reversible method of contraception, which of the following
choices can Veronica use?
A. intrauterine device C. oral contraceptive pills
B. bilateral tubal ligation D. cycle beads method

Situation - Vicky, 24 years old G2P1 on her 40 weeks AOG consulted because of hypogastric and low
back pains for the past eight hours. Physical examination showed: BP = 110/80 ; PR = 85 bpm ; FH = 32
cm ; FHB = 150 bpm. She is having her regular antenatal care at the health center and the pregnancy
seems to be normal.

66. On examination, you consider that Vicky to be on the second stage of labor because _______.
A. true bag of water ruptures C. the baby's head is crowning
B. there is lengthening of cord D. her cervix is 9 cm dilated

67. If on internal examination, the chin of the fetus lies directly posterior, therefore, the position of
the baby in the birth canal is __________.
A. mentum posterior C. mentum anterior
B. mentum transverse D. right mentum posterior

68. The structure that is located between the parietal and occipital bones of the fetal head is the ___
A. posterior fontanel C. coronal suture
B. parietal suture D. anterior fontanel

69. On internal examination the leading portion of the fetal head is 3 centimeters above the ischial
spines. This means that the presenting part is ___________.

7
A. station plus 2 C. station plus 3
B. station zero D. station minus 3

70. The following findings can be CLINICALLY evaluated during internal examination EXCEPT
__________.
A. true conjugate C. cervical effacement
B. station of the head D. cervical dilation

Situation - Nory, a 26 year old G1P1, married three months postpartum wants to know how to use the
natural methods of family planning. She intends to get pregnant after two years. She has no medical
illnesses.

71. The lactational amenorrhea method is 98% effective provided the following conditions are met.
The woman is amenorrheic and is within __________.
A. 10 months postpartum, part time breastfeeding
B. 10 months postpartum, not breastfeeding
C. 12 months postpartum, occasional breastfeeding
D. 6 months postpartum, full time breastfeeding

72. A fertile period cervical mucus can be described as _____________.


A. thick and yellow C. thin with curds
B. thin and colorless D. thick and fishy odor

73. Using the Basal Body Temperature method of natural family planning, the fertile period is
identified by a slight rise in body temperature due to the effect of the hormone ___________.
A. Estrogen C. Progesterone
B. HCG D. FSH

74. The main mechanism of action for pregnancy prevention of the cervical mucus method is ____
A. thickens cervical mucus C. makes cervical mucus thin
B. Reduces mobility of oviducts D. prevention of ovulation

75. Using the Cycle Beads method to avoid pregnancy, the menstrual cycle of the woman should be
between ________ days.
A. 28 – 34 C. 29 - 35
B. 27 – 33 D. 26 – 32

Situation - Rosario, 30 years old G3P3 gave birth a month ago. She wants to practice family planning.
She is breastfeeding her baby but still wants to have another baby girl. She is desirous of using oral
contraceptives but is concerned about its effect on breastfeeding.

76. Rosario is correct. The use combined hormonal contraceptive postpartum is not advisable. It has
a category _________ for the Medical Eligibility for contraceptive use.
A. 3 C. 2
B. 4 D. 1

77. The most appropriate method for a breastfeeding woman like her is LAM. To be very effective, it
requires the following criteria, EXCEPT _________.
A. six months postpartum C. breastfeeding every 6 hours
B. fulltime breastfeeding D. amenorrhea

78. It after three months, the patient desire to use an additional hormonal contraception, the
appropriate choice for a breastfeeding woman is __________.
A. progestin only pills C. vaginal ring
B. combined injectable contraceptives D. combined oral contraceptive

8
79. A known complication of combined oral contraceptive is _________.
A. Nausea C. loss of weight
B. Amenorrhea D. deep venous thrombosis

80. One main concern on the use of combined hormonal contraceptives postpartum is _________.
A. produces amenorrhea C. increase libido
B. lessen breastmilk secretion D. increase incidence of nausea

Situation - Mameng, 21 years old primigravida has been in labor at home for the past 6 hours before she
arrived at the municipal birthing center. History revealed that she has a 38 weeks AOG. Cervical dilation
is at 4cm, cephalic, intact bag of water.

81. The abdominal palpation findings in cephalic presentation is _________.


A. Fetal heart beat at upper abdominal quadrant C. Uterus is wider on the sides
B. Head at lower pole of uterus D. Ballotable & round mass at uterine fundus

82. A repeat exam after 3 hours showed a 7 cm cervical dilation. The next correct step to do is
__________.
A. refer to the hospital urgently C. oxytocin augmentation of labor
B. await progression of labor D. fundal push to stimulate labor

83. Continuing your labor watch, which among the following findings will make you decide to refer the
woman to the hospital?
A. Spontaneous rupture of nag of water at 9 cm dilation.
B. The plotting of cervical dilation is near the action line.
C. Cervical dilation of two centimeters for past two hours.
D. Fetal heart beat between 140-150 beats per minute

84. Which of the following findings is a sign of obstructed labor?


A. Formation of Bandl's retraction ring C. Cervix dilates one centimeter per hour
B. Fetal head is well applied to the cervix D. Bag of water breaks at 9 cm dilation

85. In case of obstructed labor due to cephalopelvic disproportion, the patient should be referred to
the hospital to undergo ___________.
A. Forceps delivery C. Episiotomy and repair
B. Cesarean section D. Vacuum extraction

Situation - Cindy, 30 years old G3P2 with a 32 weeks age of pregnancy consulted the birthing center
because of vaginal bleeding for two hours. On examination, her vital signs are: BP = 90/60, PR = 105
bpm, FH = 29 cm. Patient is slightly pale, the uterus is soft without uterine contractions.

86. Given her case history, the most likely cause of her bleeding is ____________.
A. placenta previa C. abortion
B. preterm labor D. abruptio placenta

87. The procedure that you must NOT perform in this woman is ___________.
A. monitor fetal heart rate C. take pulse rate
B. do internal examination D. monitor blood pressure

88. The MOST important life-saving procedure that you should do in this obstetric bleeding
emergency is to ____________.
A. infuse intravenous fluid C. assume Fowler's position
B. inject oxytocin D. take fetal heart rate

9
89. The procedure that can best and safely confirm your impression in this case of bleeding during
pregnancy is _____________.
A. rectal examination C. pelvic radiography
B. internal examination D. pelvic ultrasound

90. A place that is implanted at the lower uterus is a dangerous life-threatening condition because it
can result to the following maternal and fetal complication. EXCEPT ___________.
A. premature birth C. postpartum bleeding
B. pre-eclampsia D. Antepartum bleeding

Situation - Lucila, 27 years old G1P0 came to the birthing center because of active labor contractions.
She has a 39 weeks pregnancy and after ten hours of labor gave birth to a live baby girl with a birth
weight of 3000 grams. However, upon inspection of the vaginal canal, you noted some bleeding coming
from lacerations on the vagina and perineum.

91. The main blood vessels that supplies the vagina comes from the _________.
A. ovarian vein C. internal iliac vein
B. internal iliac artery D. ovarian artery

92. If the muscle __________ is torn and is not properly repair, it can lead to uncontrollable
defecation.
A. Ischiocavernous C. Transverse perineal
B. sphincter ani D. Bulbocavenous

93. The muscle of the pelvic floor that encircle the anus and is attached to the coccyx is _________.
A. Ischiocavernous C. bulbocavenous
B. sphincter ani D. transverse perineal

94. After undergoing a Professional Regulation Commission accredited training, the registered
midwife is legally allowed to perform this procedure.
A. Manual extraction placenta C. Repair second degree tear
B. Perform episiotomy D. Repair rectal lacerations

95. Based on R.A.7392, the midwife can perform the following after training, EXCEPT __________.
A. perform internal examination C. repair second degree perineal tears
B. Inject oxytocin after delivery of placenta D. routinely insert intravenous fluid in labor

Situation - Mayumi 18 years old wants to learn more about the menstrual cycle.

96. After menarche, the periodic discharge of blood, fluid and endometrial cellular debris from the
uterus is called __________.
A. Menstruation C. menarche
B. Menopause D. coitarche

97. After the menstrual bleeding, the regeneration of the new inner lining of the uterus happens to
create a proliferative endometrium due to the action of ________.
A. luteinizing hormone C. growth hormone
B. progesterone D. estrogen

98. After the proliferative phase of the following cycle, the ovulation that follows is the result of the
sudden rise of the ___________ hormone.
A. Estrogen C. growth
B. Luteinizing D. progesterone

99. If pregnancy does not happen in this cycle, the secretion of progesterone and estrogen begins to
fall because of the degeneration of the _____________.

10
A. anterior pituitary gland C. thyroid gland
B. corpus luteum D. posterior pituitary gland

100. One menstrual cycle is counted from the ___________.


A. last day of the menstrual flow to first day of the next flow
B. first day of the menstrual flow to first day of the next
C. last day of the menstrual flow to last day of next flow
D. first day of moderate menstrual flow to the next flow

FUNDAMENTALS

101. Family Planning method options for breastfeeding woman that can be used immediately
postpartum are the following, EXCEPT:
A. Female Sterilization (BTL) C. Condom
B. Intra Uterine Device (IUD) D. Progestin injection

102. It is a semi-sitting position with head of bed is raised to 45 degrees angle and convenient
for eating, reading and watching T.V.
A. Back – lying C. Reverse Trendelenburg
B. Fowler's D. Trendelenburg

103. When the tears involve the skin and vaginal mucous membrane, subcutaneous tissues,
fascia, muscles and sphincter ani and rectum is not torn, the laceration is __________ degree.
A. Second C. fourth
B. Third D. first

104. Which of following disease is NOT caused by virus?


A. Leprosy C. Measles
B. AIDS D. Hepatitis

105. Mr. Lopez has just donated blood. How many hours will be needed by his body in order
to replace the amount of blood extracted from him?
A. 8 – 10 C. 24
B. 12 D. 3 – 5

106. Some microbiologist classify this as microorganisms, but others consider these as non
living:
A. Viruses C. Bacteria
B. Protozoa D. Fungi

107. To promote rest and sleep, which of the following measures is INAPPROPRIATE?
A. Ensure appropriate lighting, temperature and ventilation
B. Decreased fluid intake 2-4 hours before sleep
C. Heavy or spicy food can be served
D. Establish a regular bed and wake-up time

108. Which of the following is a subjective symptom?


A. Cramping pain in abdomen C. Bluish discoloration of skin
B. Abdomen is firm and distended D. Client cried during interview

109. Hepatitis can be contaminated through which of the following?


1. Contaminated needles
2. Droplet spread
3. Oral and fecal secretion
A. 1, 2 & 3 C. 1 & 3 only
B. 2 & 3 only D. 3 only

11
110. After delivery, the parturient is encourage to void for at least how many hours?
A. 2 C. 3
B. 1 D. 4

111. It is the position to be assumed of the patient suffering from respiratory problem?
A. Dorsal C. Fowlers
B. Trendelenburg D. Knee-Chest

112. During the process of sterilization, which of the following is removed and destroyed?
A. Pathogens C. Non-pathogenic
B. Viruses D. Microorganism

113. A symptoms that can be directly observed is manifestation of ___________.


A. Objective C. Subjective
B. Cardinal D. Disease

114. Preventive measure of pruritus vulvae is __________.


A. proper hygiene C. flushing
B. avoid constipation D. hot sitz-bath

115. A woman who is in active labor experience severe back pain, what position will you
recommend?
A. Fowler's C. Prone
B. Dorsal recumbent D. Squatting

116. What is the rationale of removing gloves and assist patient to put on clean gown after
delivery?
A. Prevents unnecessary exposure C. Provide access to the patient
B. Provides safety D. Promotes dignity and reduces exposure

117. Heat therapy has the following useful effects, EXCEPT:


A. Dilates blood vessels C. Delayed healing of wound
B. Promotes comfort D. Increases metabolism

118. The doctor ordered STAT medication. This means to be given __________.
A. as needed by the patient C. once at a specific time
B. immediately and only once D. with or no termination date

119. The type of wound which is due to explosion, animal bite or mishandling of tools is
_________.
A. Laceration C. Avulsion
B. Abrasion D. Incision

120. Diet is one of the factors affecting sleep. Which of the following statement/s is/are true?
1. Weight loss has been associated with reduced total sleep time
2. Warm milk help people to get to sleep
3. Weight gain is associated with an increased in total sleep time

A. 2 only C. 1 and 2 only


B. 1, 2 and 3 D. 3 only

121. If after 8 hours of labor and no progress in cervical dilation but contractions are stronger
and more frequent, what is the BEST measure to be done?
A. Refer patient urgently to the hospital C. Insert IV fluids with oxytocin
B. Give medication to speed up labor D. Do I.E every 2 hours to asses dilation

12
122. Successful autoclaving means we used which of the following ?
1. 15 pound pressure
2. moist heat
3. 250 degree F
A. 2 only B. 1 and 3 only C. 1, 2 and 3 D. 1 only

123. In establishing proper referral system, which of the following is INAPROPRIATE?


A. To bring patient to your choice referral facility
B. To help prevent further deterioration of patient's condition
C. To prevent delay of transfer
D. To ensure that referral is timely

124. In doing I.E, station +2 means that the presenting part is ___________.
A. below the ischial spine C. at the vaginal os
B. above the ischial spine D. at the level with the ischial spine

125. The doctor's order is to give medication q i d. It means:


A. 4 x a day C. Every 4 hours
B. Every other day D. Twice a day

126. Effect of heat application in pain are the following, which is NOT?
A. Feeling of comfort C. Act as counter-irritant
B. Promotes relaxation D. Decrease circulation

127. Which of the family planning methods does NOT interfere with normal flow of milk?
A. IUD C. Condom
B. DMPA D. Pills

128. Pelvic-rooking exercise during pregnancy is useful to:


A. Relieve back pain C. Stretch perineal muscle
B. Make delivery easier D. Relax legs

129. A forceful separation of tissue from the body and is due to explosion, animal bite or
mishandling of tools is a type of _________ wound.
A. Contusion C. avulsion
B. Laceration D. abrasion

130. If during the fourth stage of labor, you notice that the patient is bleeding profusely and
you cannot feel the fundus, you should immediately:
A. take BP and pulse. C. gently massage the fundus.
B. refer to the hospital. D. call the doctor.

131. PAP smear should be done yearly for women at high risk. Which of the following is not
included?
A. Those who are sexually active C. Aged 18-21 years old
B. Those with multiple partners D. Those who had hysterectomy with cervix intact

132. Which of the following factors affect blood pressure when it is lowest early in the morning,
level throughout the day and peak in the later afternoon?
A. Stress C. Diurnal variation
B. Medication D. Exercise
133. The following methods can completely destroy organisms, EXCEPT:
A. Radiation C. Dessification
B. Centrifuge D. Sterilization

13
134. Promoting stomach emptying and to prevent esophageal reflex in client with hiatal hernia,
the head of bed must be raised and the foot lowered straight tilt in. This bed position is ________.
A. Reverse Trendelenburg C. Sim's
B. Fowler's D. Semi-Fowler's

135. While assisting physician in physical examination, the midwife should:


A. anticipate the doctor's needs C. keep jolly conversion with the patient
B. issue prescription to the patient D. ensure sepsis of procedure

136. Which of the following is/are reason/s why IV infusion is infused during labor?
1. It is a routine procedure
2. To hydrate patient
3. As a vehicle for medication

A. 1 & 2 only B. 3 only C. 1, 2 & 3 D. 2 & 3 only

137. Antacids can be given to pregnant women afflicted with heartburn, EXCEPT:
A. Aluminum hydroxide C. Sodium bicarbonate
B. Magnesium hydroxide D. Milk of magnesia

138. During pregnancy, which of the following is/are reason/s why woman should eat more
food during meal?
1. To help satisfy the required nutrients to stay well and strong
2. To prevent low birth weight
3. Eating more during pregnancy is NOT the cause of difficult labor

A. 3 only B. 1 & 2 only C. 1, 2 and 3 D. 2 and 3 onl

139. Which of the following guidelines is APPROPRIATE care in using bag technique?
A. Do not use anything that belong to the patient
B. Bag and its contents should be protected from contact
C. Bag can be used/placed anywhere
D. Open the bag as often as you want

140. The doctor ordered to administer medication through sublingual. Where will be the route?
A. Under the tongue C. Mouth
B. Alimentary or respiratory tract D. Mucous membrane of the cheek

141. Which of the following first aid treatment in case of ingestion of non-corrosive poison is
INAPPROPRIATE?
A. When vomiting occurs, hold the victim's face up C. Induce vomiting
B. Refer patient to the hospital if he did not vomit D. Dilute poison by giving milk or water

142. ____________ exercise for female strengthen perineal muscles thereby helps in
promoting urinary elimination.
A. Kegel C. Bicycle
B. Walking D. Stretching

143. How many weeks of pregnancy if the fetus weighs about 400 grams, fetal heart sound
are audible and the mother can feel fetal movements?
A. 20 C. 24
B. 28 D. 12
144. The following principles in splinting are appropriate, which is NOT?
A. Dress wound before splinting fracture
B. Expose the entire extremity to be splinted
C. Immobilize the joints above and below the fracture

14
D. Try to straighten fracture involving joints

145. Main reason why woman in labor with full bladder needs catheterization is to
____________.
A. helps in the rupture of membranes C. Retard fetal descent
B. prevent infection D. prevent bleeding

146. Which of the following statements about massage is INAPPROPRIATE?


A. It improve circulation of blood
B. It is not applied to the parts which are edematous
C. Heat should not be applied before the massage
D. Massage relieve discomfort due to congestion in the nerves

147. Which of the following physical examination tools in health assessment moves part of the
body being examine?
A. Palpation C. Inspection
B. Manipulation D. Percussion

148. Normally, the placenta is to be delivered within how many minutes after the delivery of
the baby?
A. 15 B. 5 C. 30 D. 40

149. Which of the following conditions is NOT a contraindication of taking body temperature
per rectum?
A. Patient with LBM C. Patient with oxygen
B. Patient with hemorrhoids D. Patient with rectal surgery

150. Which of the following expanded scope of functions of midwife is considered


malpractice?
A. Perform internal examination C. Perform episiotomy
B. Give IV infusion in an obstetrical emergency D. Suture 1st & 2nd laceration

151. Your prenatal patient came in for check-up. Her given LMP was on August 31, 2017 ,
when will be her EDC?
A. June 7, 2018 C. April 30, 2018
B. June 31, 2018 D. June 1, 2018

152. What is the duration of taking temperature per ear drum?


A. 10 minutes C. 5 seconds
B. 1 second D. 2.3 minutes

153. Measles vaccination is given in which route of administration?


A. Subcutaneous C. Intradermal
B. Intramuscular D. Oral

154. Which of the following drug/s can increase respiration?


1. Caffeine
2. Atropine
3. Morphine

A. 1, 2 & 3 B. 1 & 2 only C. 3 only D. 2 & 3 only

155. PRN medication are given on an "as needed" basis for specific signs and symptoms. It
include the following, EXCEPT:
A. Hypertension C. Cold and cough
B. Increased anxiety D. Head or earache

15
156. You want to promote stomach emptying of your patient, how will you position her?
A. Back lying C. Fowler's
B. Reverse Trendelenburg D. Semi – fowlers

157. Which of the following management of the 2nd stage of labor is the LEAST consideration
of the midwife?
A. Observance of mechanism of labor C. Close watch for 1-2 hours
B. Health teachings/educations D. Observation of powers, forces and contraction

158. Tepid sponge bath causes superficial blood vessels to ___________.


A. constrict and preserve heat C. dilate and release heat
B. dilate and preserve heat D. constrict and release heat

159. Steam inhalation is advised if our patient has cold and cough because it ____________.
A. relieves pain C. stimulate coughing
B. relieves dyspnea D. loosen non-pharyngeal secretions

160. In performing physical examination, which of the following processes assess body
movements?
A. Percussion C. Palpation
B. Auscultation D. Inspection

161. Which of the following drugs promote defecation?


A. Diuretic C. Analgesic
B. Laxative D. Expectorant

162. A white band of cloth or other material placed above a wound to stop all flow of blood is
called ________________.
A. Dressing C. tourniquet
B. Splint D. gauze

163. A primigravida came in for prenatal check up on May 20, 2014. If her LMP was on
February 26, 2014, find her AOG:
A. 11 weeks & 2 days C. 12 weeks & 1 day
B. 10 weeks & 5 days D. 11 weeks

164. Descent of the fetal head is very important because it suggest adequacy of the passage.
A midwife can evaluate it through the following, EXCEPT:
A. Abdominal palpation C. Know its station
B. Vaginal examination D. Auscultation

165. Which of the following reasons is the MOST important in taking BP of a patient?
A. For record purposes C. It is a doctor's order
B. It is a routine procedure D. To determine subsequent changes

166. To stop all flow of blood a band of cloth or rubber placed above a wound is called:
A. Tourniquet C. Dressing
B. Splint D. Gauze/plaster

167. The right amount of water in filling the hot water bag is:
A. 1/3 full C. 2/3 full
B. 1/2 full D. 1/4 full

168. The term "apnea" applies to:________

16
A. Cardiac Failure C. Pulmonary Problem
B. Respiratory Arrest D. Heart Stroke

169. It is perhaps the single most important and effective measure to prevent a nosocomial
infection and antimicrobial resistance in hospital setting:
A. Hand hygiene C. Sterile room
B. Clean surrounding D. Using facial mask

170. The following are principles about "after pains" after delivery, which is NOT?
A. It increase puerperal sepsis C. It expel blood clots
B. It promote return of the uterus to pre-gravid state D. It is usually felt after delivery

171. Which of the following comfort and relaxation measures for patient in bed is
INAPPROPRIATE?
A. Provide loose-fitting night wear C. Offer to provide back massage before sleep
B. Assist patient with hygiene routines D. Give diuretics 30 minutes before sleep

172. While the patient is on bed, what is the BEST perineal care technique?
A. down up C. sideward
B. rotating D. up down

173. The first act of thermal burn treatment is to stop the burning process. This may be
accomplished by:
A. Application of butter or shortening C. Setting cool water run over the burned area
B. Applying ice to the area D. Cold wet compresses

174. A midwife must perform the following measure/s to minimize perineal lacerations:
1. Instruct the parturient on the proper bearing down effort
2. Maintain flexion of fetal head to allow smallest diameter
3. Do Ritgen's maneuver during delivery

A. 2 & 3 only B. 2 only C. 1 only D. 1, 2 & 3

175. In smoking cigarette, the following are its harmful substances EXCEPT:
A. Tar C. Carbon Monoxide
B. Nicotine D. Narcotics

Situation - Mrs. Reyes, a 26 years old woman and 5 months pregnant came to the health center for
succeeding prenatal check-up. Based on her Home Based Mothers Record (HBMR) her obstetrical
history showed that she and her family are obese and with polyhydramnios delivery. Her tetanus toxoid
(TT) indicated that TT3 was given last year. A series of number, 2-1-1-3 was also written on her prenatal
record. Presently, she has recurrent vaginal infection.

176. How many weeks of pregnancy does the fetus weighs about 400 grams, fetal heart
sound are audible and the mother can feel fetal movements?
A. 12 C. 20
B. 16 D. 24.

177. A series of numbers is used to designate in greater detail the history of previous
pregnancy. What would 2-1-1-3 signify?
A. 2 term deliveries, 1 child living, 1 abortion, 3 premature
B. 2 term deliveries, 1 premature, 1 abortion, 3 children living
C. 2 abortion, 1 term deliveries, 1 premature, 3 children living
D. 2 children living, 1 term deliveries, 1 premature, 3 pregnancy

178. Based on her obstetrical history, what laboratory examination should be recommended?

17
A. Hepa-B Screening C. Hemoglobin/Hematocrit
B. Urinalysis D. Glucose Screening

179. Tetanus Toxoid 4 (TT4) is given at least one year after TT3. Infants born to the mother
will be protected from neonatal tetanus and will give how many years of protection for the
mother?
A. Lifetime C. 5
B. 3 D. 10

180. Return visits is very important especially for mother who are pregnant for the first time.
Which of the following is the ideal schedule?
A. Initial visit must be done as early as possible before 4 months
B. All pregnant women should have at least 8 routine antenatal visits
C. After 8th month, visit must be every month until delivery
D. Weekly visit every 6th month of pregnancy.

Situation - A 35-year old patient, Mrs. Santos came in for consultation because of cramping pain in her
abdomen and slight difficulty of breathing. As part of health care process the midwife collected and
documented client's data, followed by taking her vital signs: BP - 110/70, Pulse - 75 per minute,
Respiration - 19 per minute and was wide awake, keen and on the watch. After admission, she was
brought to the examination room to be examined by the physician.

181. Cramping pain & slight difficulty of breathing are what kind of symptoms?
A. Subjective C. Objective
B. Physical D. Clinical

182. Collecting, organizing & documenting of client's data is what part of healt care process?
A. Assessment C. Planning
B. Diagnosing D. Implement

183. Patient's pulse is 75 beats in a minute. This means that it is ____________.


A. Bounding C. irregular
B. Regular D. weak

184. Mrs. Santos level of consciousness is ____________.


A. Stupor C. comatose
B. Alert D. restless

185. With slight difficulty of breathing, the midwife must place the patient in what position?
A. Prone C. Trendelenburg
B. Supine D. Semi-fowlers

Situation - Myra Santos, 40 years old, just delivered her 5th child in a health facility. After placental
expulsion the patient bleeds. The midwife did the necessary measures to control bleeding and was able
to stopped it. After arranging all the things she gave instructions/advises to Mrs. Santos. Postpartum
continuous care was done by the midwife while patient is still in the facility.

186. What initial measure should the midwife do to control bleeding?


A. Feel the firmness of the fundus C. Measure the height of the fundus
B. Check the blood pressure D. Check the vital signs

187. After one hour postpartum, care was done by the midwife. Which of the following is NOT
appropriate?
A. Encourage patient to pass urine C. Encourage patient to eat digestible foods
B. Ensure that patient uses adult diaper D. Advise on proper postpartum care

18
188. After one day postpartum, the midwife did vulvar care. In doing so, what is the proper
direction in doing the procedure?
A. From anus to vulva C. From anterior to posterior
B. From posterior to anterior D. Side to side

189. The following are to be considered in providing postpartum perineal-genital care,


EXCEPT ______________.
A. patient is experiencing discomfort in the area C. patient has history of UTI
B. self-care abilities of the patient D. perineal-genital hygiene practices

190. The midwife should encourage breastfeeding. Which of the following practices of
breastfeeding is not appropriate?
A. Breastfeed per baby's demand
B. Water can be given while waiting for milk to come out
C. Give only colostrum to the baby
D. Use both breast alternately at each feeding

Situation - A woman who is on the second stage of labor came to your facility. With the aid of partograph
as a monitoring tool, you were able to handle her normally. Placenta was delivered completely and uterus
was contracted although with a 2nd degree laceration of the puerperium.

191. To do internal examination (I.E) you placed the patient in dorsal recumbent position
which is the BEST suitable because it _____________.
A. relaxes abdominal muscle C. helps in bearing down
B. gives comfort to the patient D. is comfortable for you

192. To provide not only privacy but also warmth to the patient, draping must be done before
doing I.E. Arrange the following according to the step-by-step procedure.
1. Prepare the client
2. Provide her privacy
3. Wash your hands
4. Explain procedure to client
5. Position and drape the patient

A. 3, 1, 5, 4, 2 C. 4, 3, 1, 2, 5
B. 5, 4, 2, 3, 1 D. 1, 5, 3, 2, 4

193. In doing I.E arrange the following procedures accordingly.


1. Wash your hands, put on sterile gloves
2. Clean the vulva with clean water & inspect it
3. Lie on her back, bending knees with leg apart
4. Explain the procedure to the patient
5. Apply lubricant on the forefinger and middle finger

A. 4, 3, 1, 2, 5 C. 1, 5, 3, 4, 2
B. 3, 1, 5, 2, 4 D. 4, 5, 2, 1, 3

194. Your findings during I.E. is station +2, which means that the presenting part is
___________.
A. above the ischial spines C. at the vaginal os
B. below the ischial spines D. level of the ischial spine
195. After placental expulsion and upon inspection you found out that there is laceration that
involve the skin and vaginal mucosa, subcutaneous tissues, fascia and muscle sphincter ani. This
means _______ degree laceration.
A. second C. first
B. fourth D. third

19
Situation - A 25 year old 6 months pregnant woman, Normie dela Cruz has the following history of
pregnancy. This is her third pregnancy with miscarriage on her first child. She already received Tetanus
Toxoid 1 and 2 on her pregnancies. Her family has history of diabetes and obesity. She is also
experiencing leg cramps.

196. Normie's parity is ______.


A. G3 P1 C. G2 P2
B. G3 P2 D. G3 P0

197. With Tetanus Toxoid (TT1) and Tetanus Toxoid 2 (TT2) given to her on previous
pregnancy, her infant will be protected from Neonatal Tetanus and Normie will be protected for
___________ years.
A. 10 C. 3
B. 5 D. 8

198. After TT1 and TT2, when is the BEST time to give TT3 so that Normie will have 5 years
of protection? It can be given at least __________ after TT2.
A. 6 months C. 1 year
B. 10 months D. 2 year

199. Based on her history, she is at low risk for gestational diabetes and should be referred for
this laboratory examination.
A. Glucose screening C. Hemoglobin determination
B. Hepa-B screening D. Urinalysis

200. Elevation of legs and adequate calcium intake are relief and preventive measures of
which of the following discomforts of pregnancy?
A. Heartburn C. Constipation
B. Back ache D. Leg cramps

INFANT CARE AND FEEDING

201. An 8 - month old infant differs from the 5 - months old in his ability to ___________.
A. smile socially C. sit securely without support
B. imitate facial expressions D. stand

202. Boy David's weight at birth is 7.7 pounds. What is his weight in kilograms?
A. 4.5 C. 3
B. 3.5 D. 4

203. Mrs. Bel reported to you that her 1-week old baby has stopped feeding well. Which of the
following is your BEST advice for her?
A. Bring baby to the health center for medical consultation
B. Shift to formula feeding temporarily
C. Feed the baby more frequently
D. Burp the baby at the middle and after breastfeeding

204. Which of the following assessment findings in the newborn indicates a need to refer the
baby to a physician?
A. Eyes with purulent discharge
B. Discharge from the nipple of a girl newborn
C. Tremors and startle movements in the neonate born 30 minutes ago
D. APGAR score of 8

20
205. Ces, 1-year old is diagnosed as having vitamin A deficiency. Which of the following foods
would you suggest to be included in her meals?
A. Grinded beef C. Mashed monggo
B. Mashed squash D. Fish fillet

206. Normally and as a measure of adequate hydration, how many times should a newborn
void daily?
A. 4 to 5 C. 2 to 3
B. 6 to 8 D. 3 to 4

207. At how many dose should measles immunization be given for infants?
A. 1 C. 2
B. 3 D. 4

208. You are assessing a newborn using the AGPAR scoring. Which of the following is done
when you are to assess the infant's color?
A. Flicking the sole of the newborn's foot. C. Observing the newborn's skin.
B. Flexing and extending the newborn's extremities. D. Auscultating the heartbeat.

209. You suspect that baby Gaby is suffering from upper respiratory infection. Hence, as you
assess the baby, about which of the following is important to ask his mother?
A. Exposure to illness C. Cleanliness of the environment
B. Play activities D. Hydration status

210. If you are assessing an infant using the APGAR scoring, which of the following are you
actually evaluating?
A. Length and weight only C. Color and respiratory rate only
B. Reflex irritability and heart rate D. Color, respiratory rate, heart rate and reflex irritability

211. Ms. Chu consulted you since her baby cries often especially during evenings and
regurgitates small amount of milk every after feeding. Which of the following will you advise Ms.
Chu?
A. Give warm water in bottle at intervals of milk feedings
B. Burp baby halfway through and after feeding
C. Breastfeed baby regularly every 3 hours
D. Alternate breastfeeding with bottle feeding especially during evenings

212. How is primary complex acquired by infants?


A. Droplets infection C. Fecal-oral route
B. Improper preparation of infant feeding D. Intake of contaminated food

213. Among breastfeeding mothers which of following is the common cause of decrease in
the amount of milk for the baby?
A. Fatigue in working mother C. Regular emptying of the breasts
B. Irregular emptying of the breasts D. Breastfeeding more frequently

214. Which of the following is a condition in infants that is characterized by extreme calorie
deficiency?
A. Marasmus C. Kwashiorkor
B. Scurvy D. Avitaminosis

215. Mrs. Jose consults you as her 7-8 weeks old baby is experiencing colic usually during
evening. Which of the following will you advise her?
A. Hold baby in an upright position while feeding
B. Temporarily shift breastfeeding to formula feeding
C. Give bottled water in between feedings

21
D. Burp the baby in between feedings/after each feeding

216. As part of your assessment you palpate neonate's anterior fontanel. Which of the
following would you consider a normal finding?
A. Depression C. Softness
B. Bulging D. Complete closure

217. In giving solid food, which of the following practices should be discouraged?
A. Wait 5-7 days before introducing another new food
B. Introduce one kind of food at a time
C. Mix solid food with milk in a feeding bottle
D. Give the solid food before breastfeeding

218. 10 - month old infant was diagnosed as having xerophthalmia. Aside from her medicines
which of the following foods will you recommend to be included in her meals?
A. Papaya & yellow vegetables C. Egg & legumes
B. Chicken breast & liver D. Fish & green leafy vegetables

219. In your home visit you noted that the baby's eyes are slightly swollen with pus
discharges. Which of the following is the BEST thing to do?
A. Demonstrate to mother how clean eyes with cooled boiled water
B. Do an eye prophylaxis
C. Advise mother to drop at least 2 drops of breastmilk every morning to baby's eyes
D. Advise mother to bring child to health center for medical consultation.

220. Which of the following assessment findings in an infant needs referral to a physician?
A. Jaundice observed on the 3rd or 4th day after birth
B. Disappearance of Moro reflex at the age of 7 weeks
C. Uncoordinated purposeless movements in the newborn
D. Cooing sounds at the age of 2 months

221. Which of the following foods is rich source of calcium for the infants?
A. Milk C. Camote Tops
B. Butter D. Chicken Breast

222. Lina, 9 months old, is diagnoses as a pneumonia case. The signs/symptoms of this
disease are:
A. convulsions, vomiting, coughs, rapid or labored breathing
B. paroxysmal coughing, dyspnea, rapid respiration, difficulty in sleeping and eating
C. shortness of breath, cough, tachypnea, episodes of wheezing
D. cough, dyspnea, tachypnea, intercostal retractions

223. Regarding techniques of breastfeeding, which of the following shows that the mother is
practicing the proper technique?
A. When the baby's upper lip is curled outwards
B. When the baby's bottom lip is curled inwards
C. When the baby is allowed to suck the nipples as long as he wants it .
D. When the baby is latched on the nipple to include more than half of the areola.

224. Which of the following assessment findings in the 24-hour old baby boy will need a
referral to the Pediatrician?
A. Jaundice C. Irregular respiration
B. Enlargement of the breast D. Audible bowel sound

22
225. At what age in months would you expect the weight of an infant to increase by twice as
much as his birth weight?
A. 10 – 11 C. 8 – 9
B. 12 – 14 D. 5 – 6

226. A mother asks the midwife if her newborn can see and hear. Which of the following
response is the most accurate?
A. "Your child can only see but no hear yet."
B. "Studies have shown that close up vision and hearing are present."
C. "It is not possible to determine this at this time."
D. "I guess the newborn can see light and hear loud sounds."

227. A 6 - month old child is for consultation. Since you see the child for the first time, which of
the following should be included in your assessment?
1. Taking general data and family history
2. Weighing the child
3. Assessing motor and social-adaptive development
A. 1, 2 and 3 B. 1 and 2 only C. 1 only D. 3 only

228. You teach mother C to recognize danger signs in her newborn. Which of these is NOT a
danger sign?
A. Spontaneous movement of the extremities C. Respiratory rate of 68 per minute
B. Stopped feeding well D. Convulsions

229. You are to assess a nine month old child. Which of the following signs will you give an
impression that the child is undernourished?
1. Night blindness
2. Swelling of face and legs
3. Loose skin folds in arms and thighs
A. 1, 2 and 3 B. 3 only C. 1 only D. 2 and 3 only

230. Which of the following explains why separation anxiety is felt by an infant?
A. Has learned that his mother is a separate individual
B. Is afraid of strangers
C. Has appreciated the presence or need for his mother
D. Has learned about object permanence

231. Lorly, 1 year old was diagnosed as having Vitamin A deficiency. Which of the following
foods would you recommend to be included on her meals?
A. fish fillet C. mashed monggo
B. mashed squash D. chopped chicken breast

232. Joy, 1 year old was hospitalized due to asthma. In order to prevent subsequent asthma
attacks, which of the following topic is MOST relevant to discuss with her parents?
A. Regular well-baby check-up.
B. Importance of personal hygiene.
C. Need for a balanced, nutritious meals for Joy.
D. Ways to get rid of known allergens at home.

233. How is measles vaccine administered?


A. Orally in three doses C. Intramuscularly in 3 doses
B. Subcutaneously in a single dose D. Intradermally in a single dose

234. A 1-year old child is being treated at home for his diarrhea. In your follow-up visit your
health teaching should be directed towards which of the following?

23
A. Prevention of dehydration
B. Preventing other children to sleep with the sick child
C. Preparation of a well-balanced diet for the child
D. Immediately washing clothing contaminated with nasal and mouth discharges

235. Which of the following statements about Vitamin A deficiency is INCORRECT?


A. Milk, liver and papaya should be part of infant's feeding to help prevent Vitamin A deficiency
B. Lack of fat in the diet contributes to poor absorption of Vitamin A
C. Presence of Bitot's spots at the sides of the eyeball is a symptom of Vitamin A deficiency
at its early stage
D. Conjunctival dryness in infant is a sign of Vitamin A deficiency

236. You were requested to attend to a sick child at home. Upon seeing the child. Which of the
following should you do first?
A. Ask mother about child's immunization C. Counsel mother on how to give home treatment
B. Classify child's illness D. Check for a danger signs

237. You just delivered a baby boy. Which of the following position will you place the newborn
in order to further drain his secretions?
A. Supine position C. Slight prone position
B. Lying flat beside his mother D. Slight Trendelenburg position

238. Which of the following is a danger sign in the newborn?


A. Respiratory rate of 80-85
B. Slightly edematous eye lids
C. Yellowish color of palms and soles appearing on the 3rd to 4th day after birth
D. Pulse rate of 100-110

239. In normal infants the voluntary active grasp activity is first evident at the age of
_________.
A. 4-5 months C. 2-3 months
B. 8-10 months D. 6-7 months

240. Which of the following permanent teeth is first to appear in infants?


A. Incisors C. Cuspid
B. Molar D. Bicuspid

241. Why is APGAR scoring done as part of newborn care?


A. To determine newborn's vital signs
B. To assess newborn's cardiac and respiratory adaptations to extrauterine life.
C. To assess newborn's reflexes
D. To check for injuries/abnormalities at birth

242. Lester, 1 year old has measles. He is communicable _____________.


A. from the first day of the appearance of the rashes up to 7 days
B. 1 to 2 days before the onset immediately after the disappearance of the rashes
C. from slightly before the beginning of the prodromal period to 4 days after appearance of
the rash.
D. during the febrile stage of the disease

243. In your home visit to a newborn you observe that the baby is suffering from lock - jaw.
Which of the following will you do?
A. Inject Vitamin K C. Resuscitate the baby
B. Ventilate the baby D. Refer the baby to the hospital

24
244. Which of the following statement show that the baby is correctly attached to his mother's
breast while feeding?
1. Lower lip is turned outwards
2. Baby's chin is touching the mother's breast
3. Sucking is slow, deep with some pauses

A. 3 only B. 2 & 3 only C. 1 & 2 only D. 1, 2 & 3

245. Boy Reno is being treated at home for his cough and fever. In your follow-up visit you
noted that he has chest indrawing. Which of the following should you advise his mother?
A. Bring Reno to the hospital for medical evaluation and treatment
B. Continue giving his medicine at home
C. Give more fluids in addition to his milk feedings
D. Feed Reno with 2 snacks aside from 3 meals a day

246. Which of the following is a sign of distress in infants?


A. Respiratory rate of 50 per minute C. Physiologic jaundice
B. Pathologic jaundice D. Cyanosis

247. Which of the following conditions will indicate that the scheduled DPT immunization of a 3
month old boy should be withheld?
A. Temperature of 39.4 °C C. Nasal congestion
B. Diarrhea D. Exposure to Varicella

248. Which of the following statements about infant's growth and development is
INCORRECT?
A. The family's socio-economic status effects growth and development
B. Environmental sanitation affects growth and development
C. Development involves growth in the infant's ability to perform some skills
D. Development proceeds from refined to gross motor skill

249. A 10 month old child is suffering from Xerophthalmia. Which of the following foods is
most appropriate to be given as supplementary feeding?
A. Mashed monggo with soup C. Mashed squash with milk
B. Mashed potato with sugar D. Egg yolk with rice

250. Which of the following reflexes can be tested by lightly touching the corner of the infant's
mouth?
A. Startle C. Tonic neck
B. Rooting D. Moro

251. Soon after a baby is born you call out the exact time of birth. What should you do next?
A. Clamp and cut the cord C. Suction the baby's nose and mouth
B. Thoroughly dry the baby D. Wrap the baby and place onto mother's breast

252. Which of the following should be avoided in the care of the newborn?
A. Bathing an hour old baby with oil.
B. Immunizing baby with BCG and HEPA B.
C. Administering erythromycin ointment to both eyes.
D. Placing the newborn on a skin-to-skin contact with his mother's breast.

253. Steve, 1 year old had a seizure attack. As part of your assessment, you will get the
following information. Which of these is of least importance?
A. Sensation noted before the onset of first attack.
B. Age at first attack
C. Any factors that might have precipitated the seizure

25
D. Birth weight, height and temperature

254. Giving of solid foods in addition to milk can be started when an infants is at the age of
A. 10 to 12 months C. 4 to 6 months
B. 7 to 10 months D. 8 to 10 months

255. A significant assessment finding in a sick child with cough is fast breathing. There are
no signs of other severe disease. Under the Integrated Management of Childhood Illness this
child can be classified as having:
A. Pneumonia C. No colds
B. No pneumonia D. Severe pneumonia

256. You explained to the students midwives who are practicing in you clinic that the
equivalent of 1 ounce of milk in grams is ____________.
A. 16.0 C. 8.0
B. 24.5 D. 28.35

257. Which of the following action by the caregiver will give a baby the feeling of safety and
security?
A. Exercising the baby's leg and arms
B. Massaging the baby
C. Carrying the baby close to the caregiver's chest while rocking him
D. Putting the baby in a motion swing designed for neonate

258. Normally, at what age in month should the startle reflex in infant's disappear?
A. 10 C. 5
B. 3 D. 8

259. Which of the following findings in the newborn will NOT need an immediate referral to a
physician?
A. Discharges from the eyes C. Too light or too heavy breathing
B. Crying excessively D. Spitting up small a amounts of milk especially after feeding

260. How is tetanus infection acquired by the newborn?


A. Through the placenta
B. Through passage at the infected birth canal
C. Through the use of unsterile articles during cord care
D. Direct contact with the birth attendant who happens to be a carrier

261. Which of the following is a fine motor skill that can be observed in the 11-12 months old
infant?
A. Picking an object with thumb and finger C. Uttering the words "bye-bye"
B. Standing by holding a furniture D. Crying when mother leaves the room

262. Which of the following observations in the newborn needs a referral to the physician?
A. Crying without tears C. Falling off of hair
B. Presence of strong grasp D. Jaundice on the first day after birth

263. A primipara consults you since she noticed a discharge from the nipples of her baby girl.
You will explain that his is due to ____________.
A. infection acquired while in the uterus.
B. poorly developed endocrine glands.
C. some maternal hormones which induces fluid secretion .
D. infection acquired during childbirth.

26
264. Boy Allen is for consultation due to fever for 2 days. He is dehydrated. Which of the
following are possible causes of dehydration in infants?
1. Vomiting
2. Diarrhea
3. Inadequate fluid intake

A. 1, 2 & 3 C. 1 & 3 only


B. 1 & 2 only D. 2 & 3 only

265. Why should APGAR Scoring be done in the newborn?


A. Have a baseline data on baby's growth and development
B. Detect presence of congenital deformities
C. Check infant's adjustment to the environment
D. Assess baby's vital functions

Situation - You work in a lying-in clinic as a staff midwife. You just delivered a baby girl. The questions
below refer to this situation:

266. Soon after birth, which of the following should you do FIRST for the baby?
A. Examine the baby and wrap in a warm blanket C. Thoroughly dry the baby
B. Suction nose and mouth as a routine procedure D. Perform APGAR scoring

267. Sixty seconds after birth you noticed that the baby has difficulty of breathing and is blue
in color. Which of the following should you do FIRST?
A. Keep the baby warm in a warm blanket and place him close to his mother.
B. Start bag-and-mask ventilation
C. Hold the baby upside down to let out secretions
D. Stimulate baby to cry by slapping both feet

268. You closely monitor the baby's condition. After 2 minutes the baby is now crying and is
pink in color. What is the next thing to do for the baby?
A. Clean the baby by wiping off the vernix
B. Check the baby's respiratory and heart rate
C. Inject Vitamin K
D. Place the baby in a skin-to-skin contact with his mother's chest

269. Within the first 3 hours after birth, Which of the following should be done for the
newborn?
A. Give the baby an oil bath and place him in a warm crib
B. Clamp and cut the umbilical cord
C. Perform eye prophylaxis
D. Weigh the baby and record his weight

270. You advise the mother to start breastfeeding once the baby shows feeding cues, Which
of the following EXCEPT ___________.
A. Crying C. Tonguing
B. Licking D. Biting fingers

Situation - You work in a birthing center. You just delivered a 6.5 pounds baby girl. You attend to the
baby's needs.
.
271. Within the first 5 to 10 seconds after birth which of the following should you do FIRST?
A. Thoroughly dry the baby using a clean dry cloth.
B. Clamp and cut the umbilical cord.
C. Wrap baby in a blanket and place beside his mother.
D. Do routine suctioning of the nose and mouth.

27
272. Within the FIRST 30 seconds after birth which of the following is important to check?
A. Malformations C. Bleeding
B. Breathing D. Birth injuries

273. As you monitor the baby's vital signs you noted that he has difficulty of breathing. Which
of the following should you do FIRST?
A. Start ventilation using bag-and-mask.
B. Refer the newborn to a Physician/ nearby hospital.
C. Place baby in skin-to-skin contact with mother's breast
D. Stimulate baby to cry by gently patting his back.

274. You observed that the newborn is cyanotic. After 2 minutes of ventilation he started
crying and is pinkish in color. No difficulty of breathing is noted. Which of the following is the
BEST action to take?
A. Place the baby in skin-to-skin contact with his mother's breast .
B. Wrap the baby with a blanket and place him in a warm crib.
C. Check baby weight, measure his length and record.
D. Place the baby in an isolette to maintain normal body.

275. You noted that the baby shows feeding cues. So you advise the mother to breastfeed her
baby. In order to ensure good positioning and attachment you discussed with her some
breastfeeding techniques. Which of these is NOT correct?
A. See to it that baby approaches breast, with lower lip to the nipple?
B. Wait until the baby's mouth is wide open before starting to feed.
C. Make sure that the baby's chin is touching mother's breast
D. Support the baby's whole body so that neck is not flexed

Situation- As a Rural Health Midwife assigned in a Barangay Health Station, you supervise the care of
infants within your catchment area.

276. In a home visit, a mother asks about toys. Which of the following will you recommends
appropriate for a 6 month old infant?
A. Rattles C. Push-pull toys
B. Squeeze toys D. Building blocks

277. You are to supervise a primipara in giving bath to her baby. Which part of the body
should be cleaned first?
A. Genitals C. Hands
B. Head D. Face

278. Regarding toys, which of the following is the MOST appropriate health teaching for a
mother of a 5 month old infant?
A. Never allow the child to play with toys alone
B. Offer toys like rattles and stacking blocks
C. Offer stuff toys as preferred by the child
D. Check toys for loose buttons and rough edges before placing this in the playpen

279. You administered DPT immunization to Baby Lyn today. You instruct her mother to bring
back Baby Lyn for her second dose of DPT after __________.
A. 3 months C. 4 weeks
B. 2 weeks D. 2 months

28
280. Mother A is concerned about the decrease in weight of her 8-day old baby. You will
explain to her that the decrease could be due to which of the following?
A. Giving of bottle feeding alternately with breast milk
B. Spitting out of milk in small amounts during feeding
C. Insufficient milk feedings
D. Excretion of fluids from the urinary bladder and bowels

Situation - Boy Lester, 11 month old is in the clinic with his mother because of pain at his left ear. You are
the midwife on duty. Guided by the Integrated Management of Childhood Illness protocol you attend to
the child.

281. Which of the following is the FIRST thing to do for Lester?


A. Examine Lester for general danger signs.
B. Ask about the duration of the ear pain.
C. Check for the presence of other symptoms.
D. Ask about Lester's immunization status

282. As part of your assessment you do the following EXCEPT ____________.


A. Ask for how long has there been ear discharge. C. Ask if Lester is able to drink.
B. Look for pus draining from the ear. D. Feel for tender swelling behind the ear.

283. Assessment findings revealed that pus is draining from the ear and discharge is reported
for more the 2 weeks. How would you classify Lester's illness?
A. Mastoiditis C. No ear infection
B. Chronic ear infection D. Acute ear infection

284. In view of the presence of ear discharge for more than 2 weeks, you can do the following
interventions but not one of these. Which of these should NOT be done?
A. Give antibiotics for 5 days. C. Treat with topical quinolone eardrops for 14 days.
B. Dry the ear by wicking. D. Immediately refer boy Lester to the hospital

285. You are to teach Lester's mother to give follow-up treatment at home. Which of the
following should you do FIRST?
A. Allow mother to give the first treatment in the clinic.
B. Instruct mother on how long treatment should be done at home.
C. Demonstrate how to give the treatment
D. Explain what is treatment and how this should be done.

Situation - Sonny, 11 months old is in the health center for consultation. He has cough for 3 to 4 days
now.
286. Having in mind the Integrated Management of Childhood Illness Guidelines, which of the
following should be done first for Sonny?
A. Classify his illness C. Conduct an assessment of his condition
B. Give initial dose of antibiotics D. Give a cough medicine to soothe his throat

287. In the case of Sonny, which of the following number of breaths per minute is considered
as fast breathing?
A. 39 C. 44
B. 36 D. 55

288. In the process of assessing Sonny, which of the following need NOT to be done?
A. Check if wheezing or stridor is present
B. Count Sonny's breath for one full minute
C. Ask mother if Sonny is able to breastfeed or drink
D. Look for chest indrawing

29
289. Assessment findings revealed that Sonny has fast breathing with chest indrawing.
How would you classify his illness?
A. Pneumonia C. Cough or Cold
B. No Pneumonia D. Severe Pneumonia

290. Sonny's illness is classified under the yellow row. Which of the following is implied by this
classification?
A. No specific treatment is needed
B. Sonny needs appropriate antibiotics
C. Sonny needs to be referred to a higher level health facility
D. No urgent measure is needed

Situation - As a midwife you are also a health educator. You help manage infant problems by educating
mothers.

291. Mrs. D consults you as she feels that her baby is not feeding well. Sometimes she is
sleepy and suckles weakly. You discuss with Mrs. D ways to wake up a sleepy baby to be able to
feed well. Which of these should be discouraged?
A. Let baby's arms and legs move by removing heavy clothing
B. Breastfeeding baby in an upright position
C. Talking to the baby while breastfeeding
D. Tapping on baby's feet

292. Mrs.G. thinks that she is not producing enough milk as her baby cries frequently. In order
to help Mrs.G, which of the following should you do FIRST?
A. Put the baby in skin-to-skin contact with mother's chest.
B. Gently massage the baby's arms, legs and back.
C. Hold the baby close and put her to her mother's breast
D. Examine the baby and observe a breastfeed.

293. Mother E has difficulty in knowing when to breastfeed and how long this should be done?
So you discuss with her signs which will show that the baby is hungry. Which of these is a late
hunger sign?
A. Opening his mouth, stretching out tongue and turning head to look for the breast.
B. Sucking on his hands / fingers or any object in contact with his mouth.
C. Increasing eye movements
D. Crying loudly, arching his back.

294. Mother H is afraid that her baby will be undernourished. She feels that she does not
produce enough breastmilk. You discuss things she can do to increase her milk
production/supply. Which of the following cannot be help to her?
A. Avoid limiting the baby's breastfeeding time in each breast.
B. Give supplementary feeding or pacifier when baby cries in between feedings.
C. Breastfeed more often and longer.
D. Improve on baby's attachment to mother's breast when feeding.

295. Mrs. J is afraid that her baby might be having hypoglycemia. Which of the following
practices can help prevent this condition?
1. Early skin-to-skin contact with mother's chest
2. Early and frequent breastfeeding
3. Avoiding water supplements

A. 2 & 3 only C. 1, 2 & 3


B. 1 & 3 only D. 2 only

30
Situation - You visited Mrs.Co, a primipara who was referred to you for follow-up. She has a 5 days old
baby. She is not aware why and how she should breastfeed her baby.

296. You explain to Mrs.Co the benefits of breastfeeding. Which of these is MOST important
for the baby?
A. Provides the baby with immunity to certain childhood disease
B. Hasten the return to normal size and shape of the uterus
C. It saves time and money
D. Facilitates mother-infant bondin

297. You guide Mrs. Co in properly positioning her baby while breastfeeding. Which of the
following is a sign that the baby is well attached to her mother's breast?
A. Presence of lip-smacking sound C. Indrawing of newborn's cheeks
B. The lower lip is pointing forward D. More areola is visible above the baby's mouth than below

298. In your second visit, you learned that the baby has difficulty in grasping her mother's
nipple. For this, which of the following will you teach Mrs.Co?
A. Express a little milk just before starting to breastfeed
B. Give milk in bottle while baby is adjusting to her mother's nipple
C. Support breast with one hand while breastfeeding
D. Try to push nipple to baby's mouth to start feeding

299. In response to Mrs. Co's question as to how often she should breastfeed her baby, which
of the following is the MOST appropriate answer?
A. Every 4 hours with cooled boiled water in between feeding
B. Depends on baby's demand for feeding
C. Every 3 to 4 hours
D. Regularly every 2 hours

300. For Mrs.Co’s, which of the following instruction on infant feeding is MOST appropriate?
A. May give rice water to supplement breast milk
B. Wake up the baby during feeding time
C. May start giving the baby cereals on the second month after birth
D. Feed newborn with breastmilk only

PRIMARY HEALTH CARE


301. Health education with emphasis on nutrition is a part of intervention done during _______
A. Prenatal check-up C. Postnatal check-up
B. Intranatal period D. Mother classes

302. The following are the steps in community organization, Except _______________.
A. Program formation C. Fact finding
B. Rehabilitation D. Needs determination

303. The following statement about Rabies is true EXCEPT:


A. Deadly bacteria C. Bite from dog, cats
B. Causes encephalitis D. Attacks the central nervous system

304. ____________ is a continuous & dynamic process of encouraging people to


understand/develop their awareness of their existing conditions by providing the skills, capability
training, and working with the people collectively & efficiently on their problems.
A. social integration C. entry phase
B. organization D. COPAR

305. The midwife involved in community development is considered expert if she


______________.

31
A. Performs her duty as a midwife C. Settle disputes
B. Identifies health problems D. Helps people achieve their goals

306. Communicable disease are the top leading causes of morbidity and mortality in the
Philippines. What services do the government (DOH) has to offer in order to minimize the
occurrence of the diseases?
A. Recruitment and additional BHWs
B. Case-finding, Treatment, Immunization, Health Education
C. More hospitals
D. Increase benefits for health worker

307. Which of the following sign and symptoms is NOT suggestive of Dengue Hemorrhagic
Fever?
A. Bleeding nose or gums, bleeding in the stools C. Black stools or vomitus
B. Warm clammy extremities D. Skin petechial

308. Maternal deaths are still high in our country. Which among the following pronouncements
based on statistics is true?
A. 10 mother die daily due to pregnancy & delivery complications
B. 13 mother die daily due to pregnancy & delivery complications
C. 15 mother die daily due to pregnancy & delivery complications
D. 16 mother die daily due to pregnancy & delivery complications

309. WHO stated that as many as 2 million death among children per year could have been
prevented by:
A. Teaching parents on what to do C. Giving quality care
B. Having a specialist to care for a sick child D. Access to immunization

310. When a women dies _______ days after termination of pregnancy from any related
cause or aggravated by her pregnancy excluding deaths from accident is called maternal death.
A. 45 C. 42
B. 28 D. 30

311. Which of the following statements defines Community Diagnosis?


A. Methods used to carry out community programs
B. Process of documenting the data about the community
C. Tool in obtaining data about the community
D. Interprets finding drawn from the profile of the community

312. Exclusively breastfeeding is the DOH mandate on newly delivered mothers. A midwife
should motivate & instruct mothers the following EXCEPT:
A. Breast milk is not always good to the baby C. Exclusively breastfeed until 6 months
B. Breastfeed on demand, 8 or more times a day D. No giving of water & vitamin

313. In calculating vaccine requirement for the year, essential information is needed EXCEPT
_____________.
A. Wastage C. Eligible targets and doses required
B. 100% buffer stock D. Expiration

314. Barangay San Roque has an eligible target population of 595 pregnant mother for 2019.
How many are expected to receive Diphtheria-Tetanus injection monthly?
A. 60 C. 49
B. 55 D. 52

315. In one of your home visits to a 3 days postpartum, you noticed that the mother has poor
appetite, irritable, headaches. She's having:

32
A. none of these C. postpartum depression
B. postpartum blues D. postpartum psychosis

316. Action plan in community organization has the following criteria Except ____________.
A. Time frame/resources needed C. Benefit
B. Objectives/activities D. Expected outcome

317. In order to determine the success or failure of programs implemented, needed data will
come from _________________.
A. demographic data C. population pyramid
B. survey D. vital statistics

318. Midwife Corazon has four under 5's children in her Barangay Health Station with
complaints of cough & diarrhea. One needs urgent attention. The following are danger signs to be
considered for referral EXCEPT __________________.
A. vomits everything C. not able to drink or feed
B. lethargic or abnormally sleepy D. stridor

319. After assessment, a 3 year old baby has chest indrawing with temperature of 38.5 °C.
Based on the IMCI program, it is classified as:
A. no pneumonia, fever C. pneumonia
B. very severe disease D. local bacterial infection

320. Baby Connie 2 years old is having watery stool 3x a day for 1 day. As a midwife, what is
the best advice to the mother?
A. give oral rehydration solution C. give lugaw
B. give buko fruit D. increase soup intake

321. ______________ is a type of epidemiology wherein survey are used to find out the
nature of the population affected by a particular disease noting the age, sex and occupation.
A. Qualitative C. Descriptive
B. Empirical D. Analytical

322. The young as well as the old are succumbing to non-communicable disease. Which
among the following is non-communicable illness?
A. Tuberculosis C. SARS
B. Malaria D. Heart disease

323. The end goal of Primary Health Care approach is for people to be:
A. assertive C. good motivator
B. self-reliant D. good leader

324. Geraldine, 2 years old, was brought to the health center. She was restless and irritable,
with sunken eyes, skin goes back slowly when pinched. Based on IMCI protocols, the baby was
having ______________.
A. Persistent diarrhea C. Severe dehydration
B. No dehydration D. Some dehydration

325. COPAR has been established in the area. Midwife Jessica has sought the support of the
barangay official to hire one barangay health worker to assist in the area. This occur during:
A. Maintenance phase C. Entry phase
B. Organization and building phase D. Maintenance & strengthening phase

326. People with the following warning signs are advised to seek consultation immediately
EXCEPT ________________.
A. Fever of 37°C & loose stool (2x a day) C. Severe Dehydration

33
B. Blood in your stool D. Severe abdominal pain or rectal pain

327. BCG immunization is usually given at birth. Midwife must reassure mother that the
following are expected, EXCEPT:
A. 2 weeks small red swelling appears
B. 5-6 weeks, small abscess appears which ulcerates
C. Apply alcohol to the site of injection incase an ulcer appears
D. Ulcer heals itself and leaves a scar

328. In organizational planning, which of the following is not considered?


A. Hire outside expert to help C. Involve the leaders in planning
B. Encourage group participation D. Inform the community about the plan

329. When a midwife organizes the community into groups to encourage active participation in
health programs/activities, she is doing __________________.
A. Program formation C. Preparation entry
B. Community mobilization D. Determination of needs

330. A 1 year old baby was assessed as having Diarrhea with some Dehydration, Cough or
Cold for four days. Based on the IMCI (integrated management of childhood illness) protocols,
treatment will be:__________
A. Treat the cough or cold C. Refer to the hospital
B. Provide treatment for cough or cold and rehydrate D. Rehydrate

331. ___________ is the unit of service in the community.


A. Family C. Barangay
B. Society D. Health Center

332. If Tetanus Toxoid 4 (TT4) is given to G4P5 pregnant, how many years will she be
protected from Tetanus?
A. 5 C. 10
B. lifetime D. 3

333. The process by which you are able to know the people through gathering of specific
profile of the community is through community ___________________.
A. Development C. assessment
B. Empowerment D. participation

334. In giving Anti-Measles the following is considered Except _________________.


A. route is subcutaneous C. clean site with cotton with alcohol
B. inject in the outer part of upper arm D. instruct mother on how to hold the baby

335. The management done to ensure the potency of vaccines from the time of manufacture,
until the time it is given to eligible target is __________________.
A. cold chain C. shake test
B. cold dog D. vaccine carrier

336. There are three (3) levels of prevention in our Philippines Health Care Delivery System.
Which among the following is considered 1st level of prevention?
A. Immunization C. Physical and respiratory therapy
B. Boiling drinking water D. Case finding

337. It is considered the universal precaution in preventing cross infection/contamination:


A. Wearing of gloves C. Use of alcohol
B. Use of hand gel D. Hand washing

34
338. Under the IMCI, a midwife can base her assessment of Severe Pneumonia if the child
has _______________.
A. Malnutrition C. high fever
B. chest indrawing D. low grade fever

339. When a midwife organizes community into groups to encourage active participation in
health programs/activities, she is doing _________________.
A. Community mobilization C. Preparation entry
B. Determination of needs D. Program formation

340. It is the time when disease are most contagious & easily transmitted to others.
A. Prodromal period C. Communicability period
B. Contact period D. Incubation period

341. A pregnant mother has received two doses of tetanus toxoid. The baby is protected
from tetanus neonatorum thru type of immunity?
A. Natural active C. Artificial active
B. Natural passive D. Artificial passive

342. The basic preparation when one intends to conduct a community assembly is:
A. Reside in the community C. Prepare agenda
B. Prepare program D. Interpret proposed program

343. The midwife as front liner in providing basic health services is usually the link between
the;
A. Local Government Unit and RHU C. Community and health team
B. Community and Local Government D. Community and RHU

344. _____________ is the process by which you are able to know the people through
gathering of specific profile of the community.
A. Community participation C. Community development
B. Community assessment D. Community empowerment

345. The study of a community clients through gathering of specific data to know the profile of
the place is:
A. Community empowerment C. Community assessment
B. Community participation D. Community development

346. Families having many children, low socio-economic status, lives in congested areas are
some characteristics of a _________________ family.
A. less fortunate C. poor
B. unhappy D. high risk

347. A midwife involved in community development is considered a therapist when she


______________.
A. Acts as a mediator to settle problems
B. Advocates support for health problems
C. Participates in activities to meet goals
D. Participates in planning action for problems of the people

348. This type of epidemiology surveys is used to find out the nature of the population affected
by a particular disease noting the age, sex, and occupation.
A. Empirical C. analytical
B. Qualitative D. descriptive

35
349. A newly diagnosed TB case will undergo free treatment at the health center. How many
months will he be treated?
A. 6 C. 4
B. 2 D. 5

350. When you advice clients on good nutrition, healthy life style, counseling in family planning
you are doing:
A. Primary prevention C. Specific protection
B. Secondary prevention D. Tertiary prevention

351. This kind of bacteria affects mainly pregnant women, newborn, elderly and adult with
impaired immune system. It is acquired by eating foods contaminated with L. Monocytogenes.
A. Listeriosis C. Typhoid
B. Salmonelliasis D. Cholera

352. An 8 months old baby, weight is 6 kgs, temp is 38.5°C is abnormally sleepy, has cough
for two days and with Chest in-drawing. How do you classify the baby's illness?
A. No pneumonia, Cough or Cold C. Pneumonia
B. Severe Pneumonia or Very Severe Disease D. Bronchitis

353. In community health care process, the first thing to be done by a midwife is ________..
A. Evaluation C. Post-conference
B. Planning intervention D. Assessment

354. When work is done through other and finished on time, the process is called ________.
A. Management C. concept
B. Supervision D. principle

355. ______________ is a type of evaluation that compares alternative care intervention in


terms of producing a give output.
A. Monitoring C. Supervision
B. Cost effective analysis D. Impact evaluatio

356. Phases of COPAR include the following EXCEPT to ______________.


A. train organized group C. recognize local communities
B. work for the people D. look for communities to serve

357. Midwife Lorna has an eligible target of 600 (0-1 year old) for the current year. If she
computes her EPI target on Anti-measles, how many vials of 10 doses will she need?
A. 120 C. 102
B. 80 D. 100

358. Based on the IMCI program protocols, three (3) rules for Diarrhea Home Therapy shall be
observed. Which among the following is NOT done?
A. Increased fluids C. Continue feeding
B. Refer when dehydration sets in D. Give IV

359. In IMCI, when a child has cough but No Pneumonia, the midwife should also assess the
following Except __________________.
A. Diarrhea C. Severe disease
B. Ear infection D. Nutrition & feeding

360. Cora, 5 years old lives in a far flung community without any drugstore was brought to the
clinic. Mother claimed he passed out 1 round worm. Midwife advised mother on the use of
herbal plant that is scientifically proven as an effective anthelminthic which is:
A. niyog-niyogan C. tsaang gubat

36
B. akapulko D. bayabas

361. A pregnant woman who receives Tetanus Toxoid 1 (TT1) immunization and no TT2 dose
after a month would mean ________________.
A. Zero percent protection C. 1 year protection
B. 3 years protection D. Baby is protected

362. What is the basic health service that aims to modify harmful practices of people and their
unscientific knowledge and attitude?
A. Maternal and Child Health C. Health Education
B. Environmental sanitation D. Communicable Disease Control Program

363. WHO protocols on the number of prenatal check is at least _________ visits.
A. 10 C. 13
B. 4 D. 8

364. A midwife has been gathering data/information through interview in her catchment areas.
She is doing _____________.
A. Community participation C. Community assembly
B. Community development D. Community assessment

365. The first action of a midwife in preparing the community for primary health care activities
is done during the :_________
A. community assembly C. Clinic activities
B. Social preparation D. home visits

Situation - Benjamin, husband of Jessica 22 years old who just delivered normally in Birthing Center two
(2) days ago went to the Health Center to refer his baby whose cord was slight bleeding as seen in the
binder used. Mother was afraid to touch the cord. What action will the midwife do?

366. After preparing her PHM bag, midwife arrived with the patient's husband. Which of the
following are to be observed first?
A. Appraise the baby C. Check on the cord
B. Do the bag technique D. Take vital signs

367. Which among the following is NOT normal to a 2 days old neonate?
A. Temperature of 37.2°C C. Slight bleeding of the cord
B. Sleeps 22-23 hours a day D. Good suck and latching

368. As a midwife, what will you advise the mother on care of the cord?
1. Do not apply anything on the cord.
2. Wash the cord with clean water in case dirt is on the stump.
3. Apply alcohol on the stump to facilitate fall off.
4. Use abdominal binder to prevent colic.

A. 1 & 4 B. 1 & 3 C. 2 & 3 D. 1 & 2

369. When cord is off and navel dry, midwife can now demonstrate full bath to mother. Which
of the following is to be observed most?
1. Do it mid-morning
2. Bath immediately after feeding
3. Take temperature before the bath
4. Avoid exposing the baby to sun

A. 1 & 3 B. 2 & 4 C. 1 & 2 D. 1 & 4

37
370. After 1 week, follow up was done by the midwife. Baby cord falls off, good suck and
latching, bowel is regular, yellowish in color, and exclusively breastfeeding. Next visit to the health
center for immunization will be __________ weeks after birth.
A. 6 C. 4
B. 8 D. 5

371. How many hours should the mother and baby stay in the facility?
A. 6 C. 12
B. 26 D. 24

372. Based on the PhilHealth Newborn Care Package, the following is done Except ______.
A. Hepatitis B & BCG C. Apply alcohol to cord
B. Newborn Screening Package D. Vit K injection

373. Route of the BCG is _______________.


A. Intradermal C. Infusion
B. Subcutaneous D. Intramuscular

374. Dosage of BCG is ______________ ml.


A. .3 C. .1
B. .05 D. .02

375. Dosage and route of Vit. K is __________ and ___________.


A. .1ml ; intramuscular C. .2ml ; subcutaneous
B. .1ml ; intradermal D. .1 ; orally

Situation - What happens to a mother and baby during labor, delivery and in the first hours after birth as a
major influence on their survival. Midwives have an important role at this time. The care they give is
critical in helping prevent complications.

376. Identify among the following the core steps which is NOT done in essential newborn care
protocols?
A. Skin to skin contact C. Immediate & thorough drying
B. Properly timed cord clamping D. Separation of the newborn from mother

377. Newborn procedures done after 90 minutes are the following EXCEPT ______________.
A. Erythromycin ointment to both eyes C. Newborn Screening Test
B. Physical assessment and weighing D. Vit K, Hepa B, BCG

378. State the dosage of Vit. K, Hepa B vaccine.


I. .1ml , .5ml
II. .2ml , .5ml
III. .3ml , .4ml
IV. .2ml , .4ml

A. IV B. I C. II D. III

379. Identify the site and route of Vit K and Hepa B vaccine.
I. Intramuscular, Vastus Lateralis
II. Intradermal, Vastus Lateralis
III. Subcutaneous, Vastus Lateralis

A. II B. I C. IV D. III

38
380. When the baby is out, you put him/her on the mother's abdomen. The baby will be in
______________ position.
A. Supine C. Lithotomy
B. Prone D. Fowler's

Situation - Tuberculosis (TB) is a chronic infectious disease cause by Mycobacterium tuberculosis,


transmitted through droplets from sputum of person with pulmonary TB while coughing or sneezing. It is
curable disease, but if left untreated can lead to disabling condition and death.

381. BCG immunization is one among the primary prevention of TB which starts at
____________.
A. anytime if expose C. Birth
B. 5 years old D. School entrants

382. The following statement about TB is true Except that it is a ____________ disease.
A. Familial C. chronic infectious
B. Virus airborne D. Curable

383. Tuberculosis could be prevented by the following Except _______________.


A. adequate home ventilation C. nutritious food
B. drinking alcoholic beverages D. limited exposure

384. People manifesting the following are considered TB symptomatic Except _________
A. cough for 2 weeks or more C. fatigue but feels strong
B. Low grade fever in the afternoon D. unexplained weight loss

385. The first thing that a midwife shall do to a person considered TB symptomatic?
A. Start treatment C. Sputum smear
B. Acid-Fast Bacillus Test (AFB) D. Refer for x-ray

Situation - Measles is highly communicable disease that is worldwide. Mortality and morbidity is higher in
developing countries. As a midwife, knowledge about its reservoir, incubation period, mode of
transmission and period of communicability helps in her health education, information activities in the
community

386. Which of the following is Not a sign of measles?


A. Cough, running nose C. Rashes starts at the lower extremities
B. Fever of 38°C x 5 days D. Red eye/ conjunctivitis

387. Select the reservoir of measles.


A. Birds C. Human
B. Cattle D. Dogs

388. Incubation period is __________ days.


A. 14 C. 21
B. 6 D. 7

389. Measles is NOT transmitted by an _____________.


A. throat secretions C. Indirect contact
B. Urine D. nasal secretion

390. How many days measles is highly communicable? Before the beginning of fever &
cough.
A. 14 C. 7
B. 21 D. 3

39
Situation - Benilda Costales 20 years old, delivered normally in the hospital three (3) days ago. She was
visited by their neighbor who is the assigned Barangay Health Worker (BHW) in the Purok. Benilda
complains of having slight fever yesterday. So the BHW referred her to the new midwife in the health
center. What will the midwife do?

391. The midwife will do the following, EXCEPT:


A. Prepare her PHM bag C. Make plan to visit
B. Check the content of her bag D. Advice the BHW to bring the mother to the health center

392. As the BHW and midwife reach the home of the patient, the midwife will:
A. Check the cord of the baby C. Open her PHM bag take the BP, temperature
B. Introduce herself and establish rapport D. Give her paracetamol tablets 500mg for fever

393. What is the most appropriate thing for the midwife to do after the bag technique
1. Appraise the mother from head to toe
2. Ask her complains and take temperature, BP, Pulse rate
3. Check lochia
4. Advice on hygiene, diet, etc.

A. 1 & 3 B. 1 & 2 C. 1 & 4 D. 2 & 3

394. Assessment result: breast engorged, temperature was 38.5°C, BP: 120/70, PR: 67bpm,
lochia was normal, no foul smell. What is the possible cause of fever?
A. Engorged breast C. Cough and colds
B. Infection D. Constipation

395. Caused of the fever was determined. Midwife appropriate advise will be:
1. Demonstrate proper latching to the mother
2. Breastfeed often at least 8 or more times a day to prevent fullness
3. Give analgesic to relieved her of the fever
4. Refer the doctor

A. 1 & 4 B. 1 & 3 C. 2 & 3 D. 1 & 2

Situation - Mrs. Barcena has 2 living children, she had one miscarriage and premature delivery. She
missed her menstrual period for three months causing her to visit the barangay health station last
February 28, 2019. LMP was November 10, 2018.

396. What will the midwife do?


A. Refer to the doctor C. Admit her the last
B. Refer to the head nurse D. Do pregnancy test

397. Based on history, her gravity and parity Mrs. Barcena is now ________________.
A. G5P2 C. G4P2
B. G3P1 D. G3P2

398. Her expected date of confinement and age of gestation will be ______________.
A. Aug. 17, 2019, AOG 14 weeks C. Aug. 17, 2019, AOG 16 weeks
B. Aug. 17, 2019, AOG 12 weeks D. Aug. 17, 2019, AOG 15 weeks

399. During the interview she complains of headache, dizziness and nausea especially in the
morning. These are _______________ signs.
A. Presumptive C. Danger
B. Positive D. Probable

40
400. Her complains will now be written in her clinic record as _______________.
A. Subjectives C. Assessment
B. Objectives D. Plans

PROFESSIONAL GROWTH & DEVELOPMENT

401. The two (2) midwives parted ways due to misunderstanding, worse, they started telling
negative things against each other in public and with other friends. The midwife filed a case
against the other midwife for destroying her reputation to the public, hence, the other midwife was
charged for ___________________.
A. Slander C. Libel
B. Perjury D. Forgery

402. The midwives enjoyment in caring for mothers and babies is a manifestation of the
bioethical principle of _____________.
A. double effect C. stewardship
B. beneficence D. totality

403. An applicant for the midwifery board examination must be a graduate midwife. The power
to confer the degree of "GM" is vested on the _______________.
A. Department of Health C. Commission on Higher Education
B. Department of Education D. Head of the respective school

404. Professional midwives must practice in accordance with their scope. Under the Midwifery
Law, which of the following is allowed?
A. Appending the title "RM" after the name while waiting for the result of the exam
B. Practicing the profession even with no license yet just observe utmost care
C. Practicing the profession using a reissued Certificate of Registration
D. Practicing using other's Certificate of Registration but with the owner's consent

405. Republic Act 7392 created the Board of Midwifery vesting thereupon the quasi-legislative
function. This means that the Board has the power to ______________.
A. administer oath to newly licensed C. investigate filed cases of midwives
B. promulgate the rules and regulations D. conduct the licensure examination

406. Which of the following is TRUE about negligence?


A. Illegal practice is a type of negligence
B. Negligence is a failure to perform due care that a prudent person would exercise
C. Negligence applies to the harm caused by the cautious performance of duties
D. Negligence is the harm unintentionally done to a patient.

407. After due process, the court found premeditation before the act was performed.
Therefore, the defendant was guilty of which of the following cases?
A. Suicide C. Homicide
B. Murder D. Parricide

408. Which of the following is NOT an expanded function of the midwife?


A. Insertion of IV Fluid during OB emergencies
B. Handling of breech deliveries
C. Suturing perineal laceration
D. Internal examination in the absence of bleeding

409. In the practice of midwifery, decision-making is one of the important roles of the midwife.
This is the reason why there is the study of:
A. ethics C. morale
B. science D. conduct

41
 
410. In a birthday party, one of the groups present were having drinking session. All of the
sudden, guests from other groups heard heated arguments from the two who were in the drinking
session. One of the two involved was saying harsh words against the other one. When there are
harsh words that are uttered to a person before a crime is committed resulting to the death of said
person, the case is considered _____________.
A. Suicide C. parricide
B. Murder D. homicide

411. Life in society is only possible by practicing the moral principles regulating human
relations this is better known as:
A. law of conscience C. human acts
B. virtue D. ethics

412. Individuals who desire to become midwives must have proper training to ensure
acquisition of competencies in performing its functions which is embodied in the
__________________.
A. midwifery curriculum C. government recognition
B. board examination D. students manual

413. To acquaint one to the different functions and duties, s/he can consult which section of
the Republic Act 7392?
A. 25 C. 23
B. 24 D. 22

414. The benefits and accountabilities of midwives working in the government are embodied in
the following, EXCEPT ________________.
A. DOH Law and Policies C. Social Security System
B. Civil Services Law and Rules D. Magna Carta for Public Health Workers

415. Before a midwife performs a procedure, there should be an assurance on the consent of
the patient. This process is the _____________.
A. confidential information C. information dissemination
B. informed consent D. provision of information

416. Before providing the resolution to a case, due process to both parties are observed. This
means that both parties _________________.
A. received the needed support from the tribunal
B. were allowed to be with their lawyer
C. have reasonable opportunities to be heard
D. presented their evidences and witnesses

417. Which of the following document is NOT needed when applying for work abroad?
A. Working visa C. Contract of service
B. Passport D. Land title

418. The function of the Board such as formulation of laws, standards and guidelines is their
____________.
A. quasi-legislative C. quasi-discretionary
B. quasi-judiciary D. quasi-ministerial

419. Foreign midwife may only practice in another country if both countries has which of the
following?
A. Special Temporary Permit C. Law of Accountability
B. Law of Reciprocity D. Profession Law

42
420. Malpractice is different from illegal practice because illegal practice is _____________.
A. practicing the profession without a license
B. performing procedures not the right way
C. putting titles after the name with no legal basis
D. deviating procedures from the correct one

421. The ethical behavior of a midwife when offering to render service to a postpartum mother
and her baby is _____________.
A. Tell the mother that you are best known midwife in the community
B. Inform the mother that you are better off in giving postpartum care than other midwives
C. Charge a professional fee that commensurate with the service rendered
D. Inform the patient that your fee is lower than the other midwives in the community

422. The midwife was found guilty by the court of misdemeanor and she was punished by ___.
A. fine and imprisonment upon court decision
B. imprisonment for a period of not less than two years but not more than seven years
C. a fine of not less Php 10,000.00 and imprisonment for a period of not less than two years but
not more than seven years
D. a fine of not less than Php 10,000.00 only

423. As a midwife, she must upheld herself by professional ethics. This means that
A. she must be guided by the principles of right action exemplary in the practice of the
profession
B. she must observe the measure stick for midwives in their professional conduct in dealing with
patients
C. her actions are based on sufficient consideration
D. her conduct must be worthy of high standards

424. In order for the midwife to become effective in performing her functions, aside from being
competent, she must be ___________.
A. Pessimistic C. religious
B. Optimistic D. close-minded

425. Which of the following statements is NOT TRUE about abortion?


A. Abortion is never permitted even if it will save the mother's life
B. Abortion is not allowed in the country based on its Constitution
C. Abortion may be allowed as per request of the family
D. Abortion is allowed as long as its objective is justifiable

426. It is not enough that a midwife has the technical skills, she must have the basic
leadership skills as well. Becoming a good leader, a midwife must possess the following,
EXCEPT ____________.
A. expert in establishing linkage with other healthcare groups
B. able to identify different channels of communications
C. capacity to recognize others weaknesses and help to improve
D. ability to encroach on other members functions.

427. One of the factors contributing to a pleasing personality is ______________.


A. Complexion C. sweet smile
B. Grooming D. good height

428. An example of an economic problem of the country is __________ that contributes to the
spread of sexually transmitted diseases.
A. Illiteracy C. urbanization
B. Prostitution D. political intervention

43
429. Conduct refers to ____________.
A. Habits C. Systematized practice of activities
B. Personal behavior D. Practice that has the force of law

430. The following are the possible positions a newly registered midwife can apply for
EXCEPT:
A. clinical instructor of a school of midwifery C. rural health midwife
B. staff midwife in the hospital D. a day-care staff midwife

431. Midwives are expected to provide care with tenderness and carefulness. However,
sometimes in spite of the efforts, accident happens. The principle of non-maleficence could be
justified through the principle of __________.
A. due care C. totality
B. beneficence D. double effect

432. A person who has completed a required course from a duly recognized government
institution and obtained the required competencies with the ultimate goal of assisting women in
pregnancy, delivery and postpartum is the _______________.
A. Caregiver C. barangay health worker
B. Midwife D. doctor

433. An untoward incident happened while the midwife is on duty. Her claim now in order to be
free from any liability is the "Doctrine of Respondent Superior". In which situation does this
doctrine apply?
A. the midwife carried an unlawful order
B. the midwife followed the lawful order
C. the midwife carried out the order of the intern
D. the midwife committed an error while on duty

434. "Unprofessional conduct" is a common complaint against midwives. This means


___________.
A. inability of the midwife to perform the assigned task
B. violation of the rules of ethical code
C. impairment of midwife's to normal functions
D. failure of the midwife to perform her task

435. Under the employee-employer relationship, a midwife could administer vaccines even if
it is not in her scope of practice. Her supervisor is answerable for her actions. This is supported
by the doctrine of _______________.
A. authority-driven C. respondent-superior
B. command-responsibility D. merit-system

436. Every barangay is required to establish a day-care center through RA __________.


A. 7192 C. 6972
B. 7610 D. 7392

437. While we are enjoying life, it is our responsibility to care for ourselves. This is our way of
practicing the bioethical principle of ___________.
A. double effect C. stewardship
B. respect to person D. totality

438. The RHU midwives work directly with the communities, they are known as;
A. tertiary level health workers C. peripheral health workers
B. second level health workers D. frontliners

44
439. The subject Ethics teaches the student midwives to arrive at ____________ every time
the encounter problems.
A. presence of mind C. plan of actions
B. decisions D. judgements

440. An effective midwife is one who has the knowledge, skills, and attitude of a professional
referred to as the ____________.
A. Confidence C. qualities
B. Competencies D. expertise

441. In order to maintain the integrity of every registered midwife, the Board of Midwifery is
tasked to ____________.
A. issue certificate of registration C. administer the licensure examination
B. uphold the laws affecting the practice D. monitor poor performing schools

442. Voluntary retirement starts at age ______________.


A. 70 C. 60
B. 65 D. 62

443. "Incompetency" is a serious problem which means _______________.


A. nonconformity to the legal scope of practice
B. want of correct knowledge and skills
C. lack of ability to discharge the required functions
D. performance of duties with utmost care

444. A midwife worked as a private duty nurse to cancer client who has operated on her action
is considered as _______________.
A. Malpractice C. Unethical
B. Negligence D. Incompetence

445. The ___________ is very instrumental in the upliftment of the profession and its practice.
A. professional organization C. professional competition
B. public opinion D. allied health profession

446. Authorities are answerable to the subordinates is an implication of the Doctrine of _____.
A. hold-over C. own-accountability
B. respondent-superior D. command-responsibility

447. Which of the following is NOT valid as reason on a petition for legal separation of
couples?
A. Non-emotional stability C. Condonement of the offense
B. Repeated physical violence D. Premeditation statements

448. The Board of Midwifery is currently revising the Midwifery Law. According to authorities, if
the Board decides to revise more than 8 provisions of the law, it is recommended to
___________.
A. suspend the current law C. amend the provisions
B. create a new law D. repeal the law

449. Which of the following competencies is NOT included during antepartum?


A. assesses progress of labor
B. conducts mothers class to pregnant women
C. performs simple laboratory exam
D. assesses accurately maternal condition during pregnancy

45
450. A profession requiring special training on normal delivery procedures, management and
care is _____________.
A. Midwifery C. delivery specialist
B. healthcare provider D. care giver

451. The midwife is presently conducting prenatal check up. If in case a patient could no
longer recall the first day of her last menstruation, her management is _________________.
A. perform leopold's maneuver
B. internal examination
C. measure the height of the fundus
D. request the patient to recall the first time she felt the baby moves in her abdomen

452. From the moment a midwife begins her professional career, she automatically assumes
an obligation to ______________.
A. become a member of the profession C. prepare herself for a greater responsibility
B. become a leader of the health team D. uphold the dignity of the profession

453. Which of the following is NOT a legal function of the registered midwife?
A. handling normal deliveries C. suturing of episiotomy
B. IV insertion during OB emergencies D. internal examination

454. Before the year 2000, registered midwives are required to undergo post-graduate training
in order to be allowed to perform the following expanded functions, EXCEPT ________________.
A. internal examination in the absence of bleeding C. handling normal deliveries
B. suturing perineal laceration D. fluid insertion during OB emergencies

455. Stories shared by a midwife according to own experiences, mentioning actual patients, is
unprofessional. However, it could be done with the ___________ of the patient.
A. Absence C. presence
B. Evidence D. consent

456. A patient can enjoy the benefits from PHILHEALTH provided she falls under what specific
case;______
A. hysterectomy, primigravida with episiotomy C. normal delivery, G4
B. mental illness, optometric services D. dental services, cosmetic surgery

457. If the midwife is required to attend a hearing to defend herself from an accusation, she
will be invited by the court through a ____________.
A. Summon C. subpoena duces tecum
B. Subpoena D. summon duces tecum

458. The mother has the right to be informed of a procedure to be done to her. Likewise, she
has the freedom to give her consent and this is evident when ______________.
A. the husband understands fully well the effects of the procedure on her wife
B. the benefits, advantages and disadvantages were explained clearly to her
C. the mother is unconscious at the time the procedure was performed to her
D. the information was manipulated when it was given to the husband

459. When you are in a situation such as attendance in an imminent delivery, your sound
decision is critical; meaning to say, you will make your decision based on the actual situation. You
have to use your ___________ function.
A. Ministerial C. executory
B. Professional D. discretionary

46
460. Professional license can be revoked due to:
A. pending case in the court C. acquittal from case
B. immorality D. failure to renew PRC ID

461. The following are the competencies of a midwife during intrapartum, EXCEPT
_____________.
A. accurate assessment of antenatal condition
B. determination of the progress of labor
C. provision of appropriate care to laboring mother
D. assessment of the mother during delivery

462. How will you manage when your friend approaches you for abortion?
A. I will advise her to look for other friends willing to help her and I will start avoiding her to have
peace of mind
B. I will refer her to another midwife who is performing the procedure but let her promise not to
implicate me in the situation
C. I will agree with her plans but it will be performed in place far from place to avoid being
implicated in her problem
D. I will convince her to continue with her pregnancy and help her realize the
consequences of her plan

463. The following are prohibited under the Midwifery Law, EXCEPT _____________.
A. practicing using a re-issued certificate of registration
B. appending RM after the name after failing the exam
C. practicing without taking the examination
D. practicing using certificate of registration of another

464. We can say that the midwife is "unprofessional" if she displays which of the following
behaviors?
A. Checks the mistakes of other midwives in front of patients
B. Discusses mistakes of other midwives in a private venue
C. Let the midwives realized their own errors
D. Ignores mistakes of others and just continue with her work

465. Midwives can participate in which of the DOH health programs as her independent
function.
A. Basic Emergency Obstetric Care C. Health Sector Reform Agenda
B. Tuberculosis Control Program D. Responsible Parenthood Program

466. Which of the following doctrines is applicable when authorities are answerable to action
of their subordinate?
A. Command-responsibility C. Accountable-partnership
B. Respondent-superior D. Hold-over

467. We can say that midwives are successful in their careers because they have _______.
A. upgrading of their profession C. the knowledge upliftment
B. the holistic growth and development D. a calling that requires coordination

468. Which of the following is NOT a ground for removal of the members of the Board?
A. Negligence C. Unprofessional conduct
B. Dishonorable conduct D. Incompetence

469. Student midwives are trained in assisting women in normal delivery. This scientific
procedures is better known as ______________.
A. Ethical C. profession
B. growth and development D. midwifery

47
470. The midwife may be exempted from liability if she failed to answer the call of patient in
labor due to flooded area, and the reason is _____________.
A. Respondent superior C. Force majeure
B. Do ut des D. Res ipsa loquitor

471. One of the qualifications of the members of the Board of Midwifery is the years of
experience in their professional practice which is _________.
A. 5 C. 7
B. 3 D. 10

472. Any information gathered by the midwife is considered confidential. If observed at all
times, then the midwife is demonstrating the bioethical principle of ______________.
A. double effect C. stewardship
B. maleficence D. beneficence

473. Midwives should not allow bottle feeding in their respective birthing clinics or else they
can be charged for violating ________________.
A. AO 26 C. RA 8981
B. RA 7392 D. EO 51

474. Midwives must be aware that right after delivery, it is their legal responsibility to
accomplish accurately and submit to PSA the ______________.
A. Birth Registration C. Birth Recognition
B. Birth Attendance D. Birth Certificate

475. We could say that a midwife has integrity if she __________.


A. Performs her legal functions C. Relies in her co-workers in doing her task
B. Submits herself to supervisor D. Practices honesty at all times

476. The law that allows midwives to perform health care and suturing of perineal laceration is;
A. RA 7392 C. RA 7160
B. RA 1161 D. RA 7277

477. For a harmonious relationship, the health team must be guided by their respective ____.
A. code of ethics C. PRC Law
B. professional laws D. personal breeding

478. Every midwife must possess a pleasing personality because this _______________.
A. means more friends C. expresses intelligence
B. contributes to fee increase D. commands respect

479. The court issues a ___________ in order for the accused to a answer to accusations
against him/her.
A. Summon C. Subpoena duces tecum
B. Witness duces tecum D. Subpoena

480. Although the midwife works independently in performing her task as member of the
health team, she should ensure that her action is in harmony with the tasks of the other members
of the health team. In this way, the patients are ensured of proper and due care. This is a
manifestation of _________________.
A. Autonomy C. nonmaleficence
B. Justice D. maleficence

Situation - The professional organizations are created to help in the upliftment of the professions, at the
same time, upliftment of the individual careers.

48
481. Aside from being healthcare providers of the community, midwives are responsible in
promoting the ideals of the profession by ______________.
A. participating in outreach programs C. conducting medical missions
B. being members of the organizations D. working hard for self-growth

482. The current PRC accredited professional organization of midwives is the ____________.
A. Midwives Foundation of the Philippines
B. Accredited Integrated Professional Organization
C. Philippine League of Private and Government Midwives
D. Integrated Midwives Association of the Philippines

483. Regardless of the organizations, midwives are assured that by joining a professional
organization, they are observing the following, EXCEPT _______________.
A. promulgating the code of technical standards of the profession
B. members of the team encroaching on other member's functions
C. enforcing the law and sanctions regarding the scope of practice
D. united as a body legally engaged in upholding the quality service in the community

484. Midwives can contribute to the success of the profession by ensuring that they observe
good breeding especially in dealing with patients by _____________.
A. knowing how to scrutinize her patients C. manifesting refined manners
B. observing her own values in work D. speaking in loud tone

485. Which of the following is NOT a characteristic of a professional midwife?


A. Discretionary but sound decisions C. Service as secondary motif
B. Autonomous D. Competent

Situation - Graduate Registered Midwives are encouraged to join the professional organizations. It is
advantageous for a midwife to become a member of the organization because she can gain lots of
benefits beside having a common goal in life and career.

486. To be qualified as member of the professional organization, a midwife needs


______________.
A. Not take active part in all activities of the profession as long as she keeps in touch with the
officers and the current trends of the profession.
B. To be registered as professional and be updated with the organization's activities and
updates relative to the profession.
C. To pass the board examination, take an oath and regularly participate in the upliftment
of the profession
D. May not take oath but regularly attends to the organization's meetings, seminars and
conventions

487. Professional organizations promote the following, EXCEPT:


A. Cohesiveness among the members
B. Establishment of a solid front in the solution of problems confronting the profession
C. Individual struggle for prestige and increase of income
D. Enhancement of knowledge and skills of midwife practitioners

488. Midwives can actively participate in researches for the improvement of the profession.
This endeavor is one of the responsibilities of the professional organizations which is embodied
with Section 11 of the _____________________.
A. Midwifery Law C. Code of Ethics
B. Modernization Act of PRC D. Expanded functions of midwives

49
489. Once a midwife becomes a member of an organization, she/he is ________________.
A. Not permitted to join other organizations
B. Obliged to comply with the provisions of the organization's by-laws
C. Judged based on the reputation of an organization
D. Forced to attend to all the activities of the organization

490. Membership to a professional organization is ________________.


A. Inherent to the functions of the organization
B. Compulsory
C. Standard Operating Procedures for graduate-registered midwives
D. Voluntary

Situation - It is the responsibility of health workers to inform patients and obtain their consent on any
procedure to be provided to the patients. Although vital, the midwife was surprised to note that the
physician on duty performed the procedure without the consent of the patient. However, the physician
was successful and the patient benefited with the action of the physician.

491. If you were the midwife faced with the problem, how would you manage the situation?
A. I will remind the physician regarding the rights of patients
B. I have nothing to do between the physician-patient relationship
C. I will just explain to the patient the necessity of the procedure
D. I will just keep silent and follow the instruction of physician

492. If ever the procedure done made the patient's condition more serious, the physician is
_____________.
A. not liable because the he/she is licensed to perform the procedure
B. liable because non-consent of the patient
C. less liable since it is done with the best of his capability
D. just in doing the procedure for the patient's benefit

493. Even if the physician was successful and the patient benefited from the action, the
_________.
A. legal implication of consent must be reviewed and be given emphasis
B. patient may still be thankful because she became better despite her not being told her actual
condition
C. physician may do the same with other patients but utmost care
D. rights of every patient sometimes are not considered as long as the physician is licensed to
perform the procedure

494. Which of the following is TRUE about patient's rights?


A. Patients' rights may be ignored if the physician is licensed and trained for the purpose
B. Patients' rights are only theories
C. Patients' rights may be considered depending on the condition and the needed procedure
D. Patients' rights must prevail at all times

495. There are situations where physicians are forced to perform procedures without patient
consent if the ____________.
A. physician was advised by the hospital chief C. patient is between life and death
B. patient has no capability to pay D. physician is experimenting a procedure

Situation - Our country is currently experiencing economic crisis. Although we are procuding more than
enough health workers, still there is inadequate supply particularly in the rural areas. Much as they want
to say in the country and serve the people, these health workers are forced to work abroad to suffice the
families' need. However, other countries are also experiencing the same situation, thus, makes it worse
for us. Some overseas Filipino health workers were sent home and some of them are still there
experiencing more serious problem.

50
496. Filipino health workers abroad who are encountering problems can best approach the
office of the __________.
A. Insurance C. Employer
B. Embassy D. Foreign Affairs

497. Filipino health workers who are sent home from other countries are called __________
A. Repatriates C. retardiates
B. Expatriates D. extradiates

498. Some Filipino health workers were sent home after facing criminal charges. The process
called ___________.
A. Extradition C. repatriation
B. Retardation D. expatriation

499. In case a Filipino health worker is displaced in other country, s/he may stay temporarily in
the care of the government because they are part of public ______________.
A. Welfare C. dominion
B. Service D. opinion

500. It is agreed by the principle that a Filipino health worker must practice and serve the
country for _________ years before s/he is allowed to work abroad.
A. 3 C. 4
B. 6 D. 5

51

You might also like